Buy BOOKS at Discounted Price

The Circle

Class 11th Mathematics RD Sharma Solution
Exercise 24.1
  1. Centre (- 2, 3) and radius 4. Find the equation of the circle with:…
  2. Centre (a, b) and radius root a^2 + b^2 . Find the equation of the circle…
  3. Centre (0, - 1) and radius 1. Find the equation of the circle with:…
  4. Centre (a cos 20 , a sin 20) and radius a. Find the equation of the circle…
  5. Centre (a, a) and radius √2 a. Find the equation of the circle with:…
  6. (x - 1)^2 + y^2 = 4 Find the centre and radius of each of the following…
  7. (x + 5)^2 + (y + 1)^2 = 9 Find the centre and radius of each of the following…
  8. x^2 + y^2 - 4x + 6y = 5 Find the centre and radius of each of the following…
  9. x^2 + y^2 - x + 2y - 3 = 0 Find the centre and radius of each of the following…
  10. Find the equation of the circle whose centre is (1, 2) and which passes through…
  11. Find the equation of the circle passing through the point of intersection of…
  12. Find the equation of the circle whose centre lies on the positive direction of…
  13. If the equations of two diameters of a circle are 2x + y = 6 and 3x + 2y = 4…
  14. Which touches both the axes at a distance of 6 units from the origin. Find the…
  15. Which touches x - axis at a distance of 5 from the origin and radius 6 units.…
  16. Which touches both the axes and passes through the point (2, 1) Find the…
  17. Passing through the origin, radius 17 and ordinate of the centre is - 15. Find…
  18. Find the equation of the circle which has its centre at the point (3, 4) and…
  19. Find the equation of the circle which touches the axes and whose centre lies on…
  20. A circle whose centre is the point of intersection of the lines 2x - 3y + 4 =…
  21. A circle of radius 4 units touches the coordinate axes in the first quadrant.…
  22. Find the equations of the circles touching y - axis at (0, 3) and making an…
  23. Find the equations of the circles passing through two points on y - axis at…
  24. If the lines 2x - 3y = 5 and 3x - 4y = 7 are the diameters of a circle of area…
  25. If the line y = √3x + k touches the circle x^2 + y^2 = 16, then find the value…
  26. Find the equation of the circle having (1, - 2) as its centre and passing…
  27. If the lines 3x - 4y + 4 = 0 and 6x - 8y - 7 = 0 are tangents to a circle,…
  28. Show that the point (x, y) given by x = 2at/1+t^2 and y = a (1-t^2/1+t^2) lies…
  29. The circle x^2 + y^2 - 2x - 2y + 1 = 0 is rolled along the positive direction…
  30. One diameter of the circle circumscribing the rectangle ABCD is 4y = x + 7. If…
  31. If the line 2x - y + 1 = 0 touches the circle at the point (2, 5) and the…
Exercise 24.2
  1. x^2 + y^2 + 6x - 8y - 24 = 0 Find the coordinates of the centre radius of each…
  2. 2x^2 + 2y^2 - 3x + 5y = 7 Find the coordinates of the centre radius of each of…
  3. 1/2 (x^2 + y^2) + xcostheta +ysintegrate heta -4 = 0 Find the coordinates of…
  4. x^2 + y^2 - ax - by = 0 Find the coordinates of the centre radius of each of…
  5. (5, 7), (8, 1) and (1, 3) Find the equation of the circle passing through the…
  6. (1, 2), (3, - 4) and (5, - 6) Find the equation of the circle passing through…
  7. (5, - 8), (- 2, 9) and (2, 1) Find the equation of the circle passing through…
  8. (0, 0), (- 2, 1) and (- 3, 2) Find the equation of the circle passing through…
  9. Find the equation of the circle which passes through (3, - 2), (- 2, 0) and has…
  10. Find the equation of the circle which passes through the points (3, 7), (5, 5)…
  11. Show that the points (3, - 2), (1, 0), (- 1, - 2) and (1, - 4) are con -…
  12. Show that the points (5, 5), (6, 4), (- 2, 4) and (7, 1) all lie on a circle,…
  13. x + y + 3 = 0, x - y + 1 = 0 and x = 3 Find the equation of the circle which…
  14. 2x + y - 3 = 0, x + y - 1 = 0 and 3x + 2y - 5 = 0 Find the equation of the…
  15. x + y = 2, 3x - 4y = 6 and x - y = 0 Find the equation of the circle which…
  16. y = x + 2, 3y = 4x and 2y = 3x Find the equation of the circle which…
  17. Prove that the centres of the three circles x^2 + y^2 - 4x - 6y - 12 = 0, x^2 +…
  18. Prove that the radii of the circles x^2 + y^2 = 1, x^2 + y^2 - 2x - 6y - 6 = 0…
  19. Find the equation of the circle which passes through the origin and cuts off…
  20. Find the equation of the circle concentric with the circle x^2 + y^2 - 6x +…
  21. Find the equation to the circle which passes through the points (1, 1)(2, 2)…
  22. Find the equation of the circle concentric with x^2 + y^2 - 4x - 6y - 3 = 0…
  23. If a circle passes through the point (0, 0), (a, 0), (0, b), then find the…
  24. Find the equation of the circle which passes through the points (2, 3) and (4,…
Exercise 24.3
  1. Find the equation of the circle, the end points of whose diameter are (2, - 3)…
  2. Find the equation of the circle the end points of whose diameter are the…
  3. The sides of a squares are x = 6, x = 9, y = 3 and y = 6. Find the equation of…
  4. Find the equation of the circle circumscribing the rectangle whose sides are x…
  5. Find the equation of the circle passing through the origin and the points where…
  6. Find the equation of the circle which passes through the origin and cuts off…
  7. Find the equation of the circle whose diameter is the line segment joining (-…
  8. The abscissae of the two points A and B are the roots of the equation x^2 + 2ax…
  9. ABCD is a square whose side is a; taking AB and AD as axes, prove that the…
  10. The line 2x - y + 6 = 0 meets the circle x^2 + y^2 - 2y - 9 = 0 at A and B.…
  11. Find the equation of the circle which circumscribes the triangle formed by the…
  12. Find the equations of the circles which pass through the origin and cut off…
Very Short Answer
  1. Write the length of the intercept made by the circle x^2 + y^2 + 2x - 4y - 5 = 0 on y -…
  2. Write the coordinates of the centre of the circle passing through (0, 0), (4, 0) and…
  3. Write the area of the circle passing through (- 2, 6) and having its centre at (1, 2).…
  4. If the abscissae and ordinates of two points P and Q are roots of the equations x^2 +…
  5. Write the equation of the unit circle concentric with x^2 + y^2 - 8x + 4y - 8 = 0.…
  6. If the radius of the circle x^2 + y^2 + ax + (1 - a) y + 5 = 0 does not exceed 5, write…
  7. Write the equation of the circle passing through (3, 4) and touching y-axis at the…
  8. If the line y = mx does not intersect the circle (x + 10)^2 + (y + 10)^2 = 180, then…
  9. Write the coordinates of the centre of the circle inscribed in the square formed by the…
Mcq
  1. If the equation of a circle is λx^2 + (2λ - 3)y^2 - 4x + 6y - 1 = 0, then the…
  2. If 2x^2 + λxy + 2y^2 + (λ - 4) x + 6y - 5 = 0 is the equation of a circle, then its…
  3. Mark the correct alternatives in each of the following : The equation x^2 + y^2 + 2x -…
  4. If the equation (4a - 3) x^2 + ay^2 + 6x - 2y + 2 = 0 represents a circle, then its…
  5. The radius of the circle represented by equation 3x^2 + 3y^2 + λxy + 9x + (λ - 6) y + 3…
  6. The number of integral values of λ for which the equation x^2 + y^2 + λx + (1 - λ) y +…
  7. The equation of the circle passing through the point (1, 1) and having two diameters…
  8. If the centroid of an equilateral triangle is (1, 1) and its one vertex is (- 1, 2),…
  9. If the point (2, k) lies outside the circles x^2 + y^2 + x - 2y - 14 = 0 and x^2 + y^2…
  10. If the point (λ, λ + 1) lies inside the region bounded by the curve x = root 25-y^2…
  11. The equation of the incircle formed by the coordinate axes and the line 4x + 3y = 6…
  12. If the circles x^2 + y^2 = 9 and x^2 + y^2 + 8y + c = 0 touch each other, then c is…
  13. If the circle x^2 + y^2 + 2ax + 8y + 16 = 0 touches x - axis, then the value of a isA.…
  14. The equation of a circle with radius 5 and touching both the coordinate axes isA. x^2…
  15. The equation of the circle passing through the origin which cuts off intercept of…
  16. The equation of the circle concentric with x^2 + y^2 - 3x + 4y - c = 0 and passing…
  17. The circle x^2 + y^2 + 2gx + 2 fy + c = 0 does not intersect x - axis, ifA. g^2 c B.…
  18. The area of an equilateral triangle inscribed in the circle x^2 + y^2 - 6x - 8y - 25 =…
  19. The equation of the circle which touches the axes of coordinates and the line x/3 +…
  20. If the circles x^2 + y^2 = a and x^2 + y^2 - 6x - 8y + 9 = 0, touch externally, then a…
  21. If (x, 3) and (3, 5) are the extremities of a diameter of a circle with centre at (2,…
  22. If (- 3, 2) lies on the circle x^2 + y^2 + 2gx + 2fy + c = 0 which is concentric with…
  23. Equation of the diameter of the circle x^2 + y^2 - 2x + 4y = 0 which passes through…
  24. Equation of the circle through origin which cuts intercepts of length a and b on axes…
  25. If the circles x^2 + y^2 + 2ax + c = 0 and x^2 + y^2 + 2by + c = 0 touch each other,…

Exercise 24.1
Question 1.

Find the equation of the circle with:

Centre ( - 2, 3) and radius 4.


Answer:

(i) Given that we need to find the equation of the circle with centre ( - 2, 3) and radius 4.



We know that the equation of the circle with centre (p, q) and radius ‘r’ is given by:


⇒ (x - p)2 + (y - q)2 = r2


Now we substitute the corresponding values in the equation:


⇒ (x - ( - 2))2 + (y - 3)2 = 42


⇒ (x + 2)2 + (y - 3)2 = 16


⇒ x2 + 4x + 4 + y2 - 6y + 9 = 16


⇒ x2 + y2 + 4x - 6y - 3 = 0


∴The equation of the circle is x2 + y2 + 4x - 6y - 3 = 0.



Question 2.

Find the equation of the circle with:

Centre (a, b) and radius .


Answer:

Given that we need to find the equation of the circle with centre (a, b) and radius .



We know that the equation of the circle with centre (p, q) and radius ‘r’ is given by:


⇒ (x - p)2 + (y - q)2 = r2


Now we substitute the corresponding values in the equation:


⇒ (x - a)2 + (y - b)2 =


⇒ x2 - 2ax + a2 + y2 - 2by + b2 = a2 + b2


⇒ x2 + y2 - 2ax - 2by = 0


∴The equation of the circle is x2 + y2 - 2ax - 2by = 0.



Question 3.

Find the equation of the circle with:

Centre (0, - 1) and radius 1.


Answer:

Given that we need to find the equation of the circle with centre (0, - 1) and radius 1.



We know that the equation of the circle with centre (p, q) and radius ‘r’ is given by:


⇒ (x - p)2 + (y - q)2 = r2


Now we substitute the corresponding values in the equation:


⇒ (x - 0)2 + (y - ( - 1))2 = 12


⇒ (x - 0)2 + (y + 1)2 = 1


⇒ x2 + y2 + 2y + 1 = 1


⇒ x2 + y2 + 2y = 0


∴The equation of the circle is x2 + y2 + 2y = 0.



Question 4.

Find the equation of the circle with:

Centre (a cos, a sin) and radius a.


Answer:

Given that we need to find the equation of the circle with centre (acosα, asinα) and radius a.



We know that the equation of the circle with centre (p, q) and radius ‘r’ is given by:


⇒ (x - p)2 + (y - q)2 = r2


Now we substitute the corresponding values in the equation:


⇒ (x - acosα)2 + (y - asinα)2 = a2


⇒ x2 - (2acosα)x + a2cos2α + y2 - (2asinα)y + a2sin2α = a2


We know that sin2θ + cos2θ = 1


⇒ x2 - (2acosα)x + y2 - 2asinαy + a2 = a2


⇒ x2 + y2 - (2acosα)x - (2asinα)y = 0


∴The equation of the circle is x2 + y2 - (2acosα)x - (2asinα)y = 0.



Question 5.

Find the equation of the circle with:

Centre (a, a) and radius √2 a.


Answer:

Given that we need to find the equation of the circle with centre (a, a) and radius √2a.



We know that the equation of the circle with centre (p, q) and radius ‘r’ is given by:


⇒ (x - p)2 + (y - q)2 = r2


Now we substitute the corresponding values in the equation:


⇒ (x - a)2 + (y - a)2 = (a)2


⇒ x2 - 2ax + a2 + y2 - 2ay + a2 = 2a2


⇒ x2 + y2 - 2ax - 2ay = 0


∴The equation of the circle is x2 + y2 - 2ax - 2ay = 0.



Question 6.

Find the centre and radius of each of the following circles:

(x - 1)2 + y2 = 4


Answer:

(i) Given that we need to find the centre and radius of the given circle (x - 1)2 + y2 = 4.



We know that the standard equation of a circle with centre (a, b) and having radius ‘r’ is given by:


⇒ (x - a)2 + (y - b)2 = r2 - - - - - (1)


Let us convert given circle’s equation into the standard form.


⇒ (x - 1)2 + y2 = 4


⇒ (x - 1)2 + (y - 0)2 = 22 ..... - (2)


Comparing (2) with (1), we get


⇒ Centre = (1, 0) and radius = 2


∴The centre and radius of the circle is (1, 0) and 2.



Question 7.

Find the centre and radius of each of the following circles:

(x + 5)2 + (y + 1)2 = 9


Answer:

Given that we need to find the centre and radius of the given circle (x + 5)2 + (y + 1)2 = 9.



We know that the standard equation of a circle with centre (a, b) and having radius ‘r’ is given by:


⇒ (x - a)2 + (y - b)2 = r2 - - - - - (1)


Let us convert given circle’s equation into the standard form.


⇒ (x + 5)2 + (y + 1)2 = 9


⇒ (x - ( - 5))2 + (y - ( - 1))2 = 32 - - - - (2)


Comparing (2) with (1), we get


⇒ Centre = ( - 5, - 1) and radius = 3


∴The centre and radius of the circle is ( - 5, - 1) and 3.



Question 8.

Find the centre and radius of each of the following circles:

x2 + y2 - 4x + 6y = 5


Answer:

Given that we need to find the centre and radius of the given circle x2 + y2 - 4x + 6y = 5.



We know that the standard equation of a circle with centre (a, b) and having radius ‘r’ is given by:


⇒ (x - a)2 + (y - b)2 = r2 - - - - - (1)


Let us convert given circle’s equation into the standard form.


⇒ x2 + y2 - 4x + 6y = 5


⇒ (x2 - 4x + 4) + (y2 + 6y + 9) = 5 + 4 + 9


⇒ (x - 2)2 + (y + 3)2 = 18


⇒ (x - 2)2 + (y - ( - 3))2 = (3)2 - - - (2)


Comparing (2) with (1), we get


⇒ Centre = (2, - 3) and radius = 3


∴The centre and radius of the circle is (2, - 3) and 3.



Question 9.

Find the centre and radius of each of the following circles:

x2 + y2 - x + 2y - 3 = 0


Answer:

Given that we need to find the centre and radius of the given circle x2 + y2 - x + 2y - 3 = 0.



We know that the standard equation of a circle with centre (a, b) and having radius ‘r’ is given by:


⇒ (x - a)2 + (y - b)2 = r2 - - - - - (1)


Let us convert given circle’s equation into the standard form.


⇒ x2 + y2 - x + 2y - 3 = 0




- - - - - (2)


Comparing (2) with (1), we get


⇒ Centre = (, - 1) and radius =


∴The centre and radius of the circle is (, - 1) and.



Question 10.

Find the equation of the circle whose centre is (1, 2) and which passes through the point (4, 6).


Answer:

Given that we need to find the equation of the circle whose centre is (1, 2) and passing through the point (4, 6).



We know that the radius of the circle is the distance between the centre and any point on the radius. So, we find the radius of the circle.


We know that the distance between the two points (x1,y1) and (x2,y2) is .


Let us assume r is the radius of the circle.






⇒ r = 5 units ..... (1)


We know that the equation of the circle with centre (p, q) and having radius ‘r’ is given by:


⇒ (x - p)2 + (y - q)2 = r2


Now we substitute the corresponding values in the equation:


⇒ (x - 1)2 + (y - 2)2 = 52


⇒ x2 - 2x + 1 + y2 - 4y + 4 = 25


⇒ x2 + y2 - 2x - 4y - 20 = 0.


∴The equation of the circle is x2 + y2 - 2x - 4y - 20 = 0.



Question 11.

Find the equation of the circle passing through the point of intersection of the lines x + 3y = 0 and 2x – 7y = 0 and whose centre is the point of intersection of the lines x + y + 1 = 0 and x – 2y + 4 = 0.


Answer:

Given that the circle has the centre at the intersection point of the lines x + y + 1 = 0 and x - 2y + 4 = 0 and passes through the point of intersection of the lines x + 3y = 0 and 2x - 7y = 0.



Let us find the points of intersection of the lines.


On solving the lines x + 3y = 0 and 2x - 7y = 0, we get the point of intersection to be (0, 0)


On solving the lines x + y + 1 and x - 2y + 4 = 0, we get the point of intersection to be ( - 2, 1)


We have circle with centre ( - 2,1) and passing through the point (0,0).


We know that the radius of the circle is the distance between the centre and any point on the radius. So, we find the radius of the circle.


We know that the distance between the two points (x1,y1) and (x2,y2) is .


Let us assume r is the radius of the circle.





⇒ r = √5 ..... (1)


We know that the equation of the circle with centre (p, q) and having radius ‘r’ is given by:


⇒ (x - p)2 + (y - q)2 = r2


Now we substitute the corresponding values in the equation:


⇒ (x - ( - 2))2 + (y - 1)2 = ()2


⇒ (x + 2)2 + (y - 1)2 = 5


⇒ x2 + 4x + 4 + y2 - 2y + 1 = 5


⇒ x2 + y2 + 4x - 2y = 0.


∴The equation of the circle is x2 + y2 + 4x - 2y = 0.



Question 12.

Find the equation of the circle whose centre lies on the positive direction of y - axis at a distance 6 from the origin and whose radius is 4.


Answer:

Given that we need to find the equation of the circle whose centre lies on the positive y - axis at a distance of 6 from the origin and having radius 4.



Since the centre lies on the positive y - axis at a distance of 6 from the origin, we get the centre (0, 6).


We have a circle with centre (0, 6) and having radius 4.


We know that the equation of the circle with centre (p, q) and having radius ‘r’ is given by:


⇒ (x - p)2 + (y - q)2 = r2


Now we substitute the corresponding values in the equation:


⇒ (x - 0)2 + (y - 6)2 = 42


⇒ x2 + y2 - 12y + 36 = 16


⇒ x2 + y2 - 12y + 20 = 0.


∴The equation of the circle is x2 + y2 - 12y + 20 = 0.



Question 13.

If the equations of two diameters of a circle are 2x + y = 6 and 3x + 2y = 4 and the radius is 10, find the equation of the circle.


Answer:

Given that the circle has the radius 10 and has diameters 2x + y = 6 and 3x + 2y = 4.



We know that the centre is the intersection point of the diameters.


On solving the diameters, we get the centre to be (8, - 10).


We have a circle with centre (8, - 10) and having radius 10.


We know that the equation of the circle with centre (p, q) and having radius ‘r’ is given by:


⇒ (x - p)2 + (y - q)2 = r2


Now we substitute the corresponding values in the equation:


⇒ (x - 8)2 + (y - ( - 10))2 = 102


⇒ (x - 8)2 + (y + 10)2 = 100


⇒ x2 - 16x + 64 + y2 + 20y + 100 = 100


⇒ x2 + y2 - 16x + 20y + 64 = 0.


∴The equation of the circle is x2 + y2 - 16x + 20y + 64 = 0.



Question 14.

Find the equation of the circle

Which touches both the axes at a distance of 6 units from the origin.


Answer:

Given that we need to find the equation of the circle which touches both the axes at a distance of 6 units from the origin.



A circle touches the axes at the points (±6, 0) and (0,±6).


From the figure, we can see that the centre of the circle is (±6,±6).


We have a circle with centre (±6,±6) and passing through the point (0,6).


We know that the radius of the circle is the distance between the centre and any point on the radius. So, we find the radius of the circle.


We know that the distance between the two points (x1,y1) and (x2,y2) is .


Let us assume r is the radius of the circle.




⇒ r = √36


⇒ r = 6 ..... (1)


We know that the equation of the circle with centre (p, q) and having radius ‘r’ is given by:


⇒ (x - p)2 + (y - q)2 = r2


Now we substitute the corresponding values in the equation:


⇒ (x±6)2 + (y±6)2 = ()2


⇒ x2±12x + 36 + y2±12y + 36 = 36


⇒ x2 + y2±12x±12y + 36 = 0.


∴The equation of the circle is x2 + y2±12x±12y + 36 = 0. We get four circles which satisfy this condition.



Question 15.

Find the equation of the circle

Which touches x - axis at a distance of 5 from the origin and radius 6 units.


Answer:

Given that we need to find the equation of the circle which touches x - axis at a distance of 5 units from the origin and having radius 6 units.



A circle touches the x - axis at the points (±5, 0).


Let us assume the centre of the circle is (±5, a).


We have a circle with centre (5, a) and passing through the point (5,0) and having radius 6.


We know that the radius of the circle is the distance between the centre and any point on the radius. So, we find the radius of the circle.


We know that the distance between the two points (x1,y1) and (x2,y2) is .


Let us assume r is the radius of the circle.




⇒ 6 = √(a2 )


⇒ |a| = 6


⇒ a = ±6 ..... (1)


We have got the centre at (±5, ±6) and having radius 6 units.


We know that the equation of the circle with centre (p, q) and having radius ‘r’ is given by:


⇒ (x - p)2 + (y - q)2 = r2


Now we substitute the corresponding values in the equation:


⇒ (x ± 5)2 + (y ± 6)2 = (6)2


⇒ x2 ± 10x + 25 + y2 ± 12y + 36 = 36


⇒ x2 + y2 ± 10x ± 12y + 25 = 0.


∴The equation of the circle is x2 + y2 ± 10x ± 12y + 25 = 0. We get four circles which satisfy this condition.



Question 16.

Find the equation of the circle

Which touches both the axes and passes through the point (2, 1)


Answer:

Given that we need to find the equation of the circle which touches both axes and passes through the point (2, 1).



Let us assume the circle touches the x-axis at the point (a, 0) and y-axis at the point (0,a).


Then we get to the centre of the circle as (a, a).


We have a circle with centre (a, a) and passing through the point (2, 1) and having radius |a|.


We know that the radius of the circle is the distance between the centre and any point on the radius. So, we find the radius of the circle.


We know that the distance between the two points (x1,y1) and (x2,y2) is .


Let us assume r is the radius of the circle.



⇒ a2 = a2 - 4a + 4 + a2 - 2a + 1


⇒ a2 - 6a + 5 = 0


⇒ a2 - 5a - a + 5 = 0


⇒ a(a - 5) - 1(a - 5) = 0


⇒ (a - 1)(a - 5) = 0


⇒ a - 1 = 0 or a - 5 = 0


⇒ a = 1 or a = 5 ..... (1)


Case (i)


We have got the centre at (5, 5) and having radius 5 units.


We know that the equation of the circle with centre (p, q) and having radius ‘r’ is given by:


⇒ (x - p)2 + (y - q)2 = r2


Now we substitute the corresponding values in the equation:


⇒ (x - 5)2 + (y - 5)2 = 52


⇒ x2 - 10x + 25 + y2 - 10y + 25 = 25


⇒ x2 + y2 - 10x - 10y + 25 = 0.


∴The equation of the circle is x2 + y2 - 10x - 10y + 25 = 0.


Case (ii)


We have got the centre at (1, 1) and having a radius 1 unit.


We know that the equation of the circle with centre (p, q) and having radius ‘r’ is given by:


⇒ (x - p)2 + (y - q)2 = r2


Now we substitute the corresponding values in the equation:


⇒ (x - 1)2 + (y - 1)2 = 12


⇒ x2 - 2x + 1 + y2 - 2y + 1 = 1


⇒ x2 + y2 - 2x - 2y + 1 = 0


∴ The equation of the circle is x2 + y2 - 2x - 2y + 1 = 0.



Question 17.

Find the equation of the circle

Passing through the origin, radius 17 and ordinate of the centre is - 15.


Answer:

Given that we need to find the equation of the circle which passes through the origin, having radius 17 units and ordinate of the centre is - 15.



Let us assume the abscissa of the centre be a then we get to the centre of the circle as (a, - 15).


We have a circle with centre (a, - 15) and passing through the point (0, 0) and having radius 17.


We know that the radius of the circle is the distance between the centre and any point on the radius. So, we find the radius of the circle.


We know that the distance between the two points (x1,y1) and (x2,y2) is .


Let us assume r is the radius of the circle.



⇒ 172 = a2 + ( - 15)2


⇒ 289 = a2 + 225


⇒ a2 = 64


⇒ |a| = √64


⇒ |a| = 8


⇒ a = ±8 ..... (1)


We have got the centre at (±8, - 15) and having radius 17 units.


We know that the equation of the circle with centre (p, q) and having radius ‘r’ is given by:


⇒ (x - p)2 + (y - q)2 = r2


Now we substitute the corresponding values in the equation:


⇒ (x±8)2 + (y - 15)2 = 172


⇒ x2±16x + 64 + y2 - 30y + 225 = 289


⇒ x2 + y2±16x - 30y = 0.


∴The equation of the circle is x2 + y2±16x - 30y = 0.



Question 18.

Find the equation of the circle which has its centre at the point (3, 4) and touches the straight line 5x + 12y – 1 = 0.


Answer:

Given that we need to find the equation of the circle with centre (3, 4) and touches the straight line 5x + 12y - 1 = 0.



Since the circle touches the line at a single point and the circle passes through that point.


We know that the radius of a circle is the distance between the centre and any point that lies on the circle.


Here the point lies on the circle and also on the line, the distance between the points is equal to the perpendicular distance from the centre on to the line 5x + 12y - 1 = 0.


We know that the perpendicular distance from the point (x1,y1) on to the line ax + by + c = 0 is given by .


Let us assume ‘r’ be the radius of the circle.





.


We have a circle with centre (3, 4) and having a radius .


We know that the equation of the circle with centre (p, q) and having radius ‘r’ is given by:


⇒ (x - p)2 + (y - q)2 = r2


Now we substitute the corresponding values in the equation:




⇒ 169x2 + 169y2 - 1014x - 1352y + 4225 = 3844


⇒ 169x2 + 169y2 - 1014x - 1352y + 381 = 0.


∴The equation of the circle is 169x2 + 169y2 - 1014x - 1352y + 381 = 0.



Question 19.

Find the equation of the circle which touches the axes and whose centre lies on x – 2y = 3.


Answer:

We need to find the equation of the circle the axes and centre lies on x - 2y = 3.



Let us assume the circle touches the axes at (a,0) and (0,a) and we get the radius to be |a|.


We get the centre of the circle as (a, a). This point lies on the line x – 2y = 3


⇒ a - 2(a) = 3


⇒ - a = 3


⇒ a = - 3


Centre = (a, a) = ( - 3, - 3) and radius of the circle(r) = | - 3| = 3


We have circle with centre ( - 3, - 3) and having radius 3.


We know that the equation of the circle with centre (p, q) and having radius ‘r’ is given by:


⇒ (x - p)2 + (y - q)2 = r2


Now we substitute the corresponding values in the equation:


⇒ (x - ( - 3))2 + (y - ( - 3))2 = 32


⇒ (x + 3)2 + (y + 3)2 = 9


⇒ x2 + 6x + 9 + y2 + 6y + 9 = 9


⇒ x2 + y2 + 6x + 6y + 9 = 0.


∴The equation of the circle is x2 + y2 + 6x + 6y + 9 = 0.



Question 20.

A circle whose centre is the point of intersection of the lines 2x – 3y + 4 = 0 and 3x + 4y – 5 = 0 passes through the origin. Find its equation.


Answer:

Given that the circle has the centre at the intersection point of the lines 2x - 3y + 4 = 0 and 3x + 4y - 5 = 0 and passes through the origin.



Let us find the points of intersection of the lines.


On solving the lines 2x - 3y + 4 = 0 and 3x + 4y - 5 = 0, we get the point of intersection to be


We have circle with centre and passing through the point (0,0).


We know that radius of the circle is the distance between the centre and any point on the radius. So, we find the radius of the circle.


We know that the distance between the two points (x1,y1) and (x2,y2) is .


Let us assume r is the radius of the circle.






..... (1)


We know that the equation of the circle with centre (p, q) and having radius ‘r’ is given by:


⇒ (x - p)2 + (y - q)2 = r2


Now we substitute the corresponding values in the equation:





⇒ 17x2 + 17y2 + 2x - 44y = 0.


∴The equation of the circle is 17x2 + 17y2 + 2x - 44y = 0.



Question 21.

A circle of radius 4 units touches the coordinate axes in the first quadrant. Find the equations of its images with respect to the line mirrors x = 0 and y = 0.


Answer:

Given that the circle having radius 4 units touches the coordinate axes in the first quadrant.



Let us assume the circle touches the co - ordinate axes at (a,0) and (0,a). Then the circle will have the centre at (a, a) and radius |a|.


It is given that the radius is 4 units. Since the circle touches the axes in the first quadrant, we will have the centre in the first quadrant.


The centre of the circle is (4, 4).


The centres of the mirrors of the circle w.r.t x = 0 and y = 0 is ( - 4, 4) and (4, - 4).


Case (i):


We have a circle with centre ( - 4, 4) and having radius 4.


We know that the equation of the circle with centre (p, q) and having radius ‘r’ is given by:


⇒ (x - p)2 + (y - q)2 = r2


Now we substitute the corresponding values in the equation:


⇒ (x - ( - 4))2 + (y - 4)2 = 42


⇒ (x + 4)2 + (y - 4)2 = 16


⇒ x2 + 8x + 16 + y2 - 8y + 16 = 16


⇒ x2 + y2 + 8x - 8y + 16 = 0.


∴The equation of the circle is x2 + y2 + 8x - 8y + 16 = 0.


Case (ii):


We have circle with centre (4, - 4) and having radius 4.


We know that the equation of the circle with centre (p, q) and having radius ‘r’ is given by:


⇒ (x - p)2 + (y - q)2 = r2


Now we substitute the corresponding values in the equation:


⇒ (x - 4)2 + (y - ( - 4))2 = 42


⇒ (x - 4)2 + (y + 4)2 = 16


⇒ x2 - 8x + 16 + y2 + 8y + 16 = 16


⇒ x2 + y2 - 8x + 8y + 16 = 0.


∴The equation of the circle is x2 + y2 - 8x + 8y + 16 = 0.



Question 22.

Find the equations of the circles touching y - axis at (0, 3) and making an intercept of 8 units on the x - axis.


Answer:

Given that the circle is touching the y - axis at (0,3) and making an intercept of 8 units on the x - axis.



Let us assume the circle intersects x - axis at the points A, B. Then the length of AB = 8units.


Let us assume ‘O’ be the centre of the circle and ‘M’ be the mid - point of the line AB.


Since circle touches y - axis at (0,3), we assume the centre of the circle be (h,3).


From the figure we get OM = 3 units and AM = 4units. The points AMO forms a right angled triangle. We have AO as radius.


⇒ AO2 = OM2 + AM2


⇒ AO2 = 32 + 42


⇒ AO2 = 9 + 16


⇒ AO2 = 25



⇒ AO = 5


We have circle with centre (h,3), passing through the point (0,3) and having radius 5 units.


We know that radius of the circle is the distance between the centre and any point on the radius. So, we find the radius of the circle.


We know that the distance between the two points (x1,y1) and (x2,y2) is .


Let us assume r is the radius of the circle.




⇒ 5 = |h|


⇒ h = ±5 ..... (1)


We have circle with centre (±5,3) and having radius 5 units.


We know that the equation of the circle with centre (p, q) and having radius ‘r’ is given by:


⇒ (x - p)2 + (y - q)2 = r2


Now we substitute the corresponding values in the equation:


⇒ (x±5)2 + (y - 3)2 = 52


⇒ x2±10x + 25 + y2 - 6y + 9 = 25


⇒ x2 + y2±10x - 6y + 9 = 0


∴The equations of the circles is x2 + y2±10x - 6y + 9 = 0.



Question 23.

Find the equations of the circles passing through two points on y - axis at distances 3 from the origin and having radius 5.


Answer:

Given that we need to find the equations of the circles passing through the points on y - axis at distances 3 from origin and having radius 5.



Since the points are 3 units away from origin on the y - axis, the points will be (0,3) and (0, - 3).


Let us assume the centre of this circle be (h,k).


We know that the equation of the circle with centre (p,q) and having radius ‘r’ is given by:


⇒ (x - p)2 + (y - q)2 = r2


Now we substitute the corresponding values in the equation:


⇒ (x - h)2 + (y - k)2 = 52


⇒ (x - h)2 + (y - k)2 = 25 ..... (1)


Since circle passes through the point (0,3). We substitute this point in eq(1)


⇒ (0 - h)2 + (3 - k)2 = 25


⇒ h2 + (3 - k)2 = 25 ..... - - (2)


Since circle passes through the point (0, - 3). We substitute this point in eq(1)


⇒ (0 - h)2 + ( - 3 - k)2 = 25


⇒ h2 + (3 + k)2 = 25 ..... - - (3)


On solving (2) and (3), we get


⇒ h = ±4 and k = 0


We have circle with centre (±4,0) and having radius 5 units.


We know that the equation of the circle with centre (p, q) and having radius ‘r’ is given by:


⇒ (x - p)2 + (y - q)2 = r2


Now we substitute the corresponding values in the equation:


⇒ (x±4)2 + (y - 0)2 = 52


⇒ x2±8x + 16 + y2 = 25


⇒ x2 + y2±8x - 9 = 0


∴The equations of the circles is x2 + y2±8x - 9 = 0.



Question 24.

If the lines 2x – 3y = 5 and 3x – 4y = 7 are the diameters of a circle of area 154 squares units, then obtain the equation of the circle.


Answer:

Given that we need to find the equation of the circle which has diameters 2x - 3y = 5 and 3x - 4y = 7 and having area 154 square units.



We know that the centre of the circle is the point of intersection of the diameters.


On solving the diameters, we get the centre to be (1, - 1).


We know that the area of the circle is given by r2, where r is the radius of the circle.





⇒ r = 7


We have a circle with centre (1, - 1) and having radius 7 units.


We know that the equation of the circle with centre (p, q) and having radius ‘r’ is given by:


⇒ (x - p)2 + (y - q)2 = r2


Now we substitute the corresponding values in the equation:


⇒ (x - 1)2 + (y - ( - 1))2 = 72


⇒ (x - 1)2 + (y + 1)2 = 49


⇒ x2 - 2x + 1 + y2 + 2y + 1 = 49


⇒ x2 + y2 - 2x + 2y - 47 = 0


∴The equations of the circles is x2 + y2 - 2x + 2y - 47 = 0.



Question 25.

If the line y = √3x + k touches the circle x2 + y2 = 16, then find the value of k.


Answer:

Given that the line y = √3x + k touches the circle x2 + y2 = 16.



Here the circle has centre at (0, 0) and radius 4.


The line touches the circle at a point. So, the distance between this point and the centre is equal to the radius of the circle.


This distance is the same as the perpendicular distance between the centre and the line.


We know that the perpendicular distance from the point (x1,y1) to the line ax + by + c = 0 is given by .





⇒ |k| = 8


⇒ k = ±8


∴The value of k is ±8.



Question 26.

Find the equation of the circle having (1, - 2) as its centre and passing through the intersection of the lines 3x + y = 14 and 2x + 5y = 18.


Answer:

Given that we need to find the equation of the circle with centre (1, - 2) and passing through the point of intersection of the lines 3x + y = 14 and 2x + 5y = 18.



Let us first find the point of intersection of lines 3x + y = 14 and 2x + 5y = 18.


On solving the lines, we get the intersection point to be (4,2).


We have a circle with centre (1, - 2) and passing through the point (4,2).


We know that the radius of the circle is the distance between the centre and any point on the radius. So, we find the radius of the circle.


We know that the distance between the two points (x1,y1) and (x2,y2) is .


Let us assume r is the radius of the circle.





⇒ r = √25


⇒ r = = 5 ..... (1)


We know that the equation of the circle with centre (p, q) and having radius ‘r’ is given by:


⇒ (x - p)2 + (y - q)2 = r2


Now we substitute the corresponding values in the equation:


⇒ (x - 1)2 + (y - ( - 2))2 = (5)2


⇒ (x - 1)2 + (y + 2)2 = 25


⇒ x2 - 2x + 1 + y2 + 4y + 4 = 25


⇒ x2 + y2 - 2x + 4y - 20 = 0.


∴The equation of the circle is x2 + y2 - 2x + 4y - 20 = 0.



Question 27.

If the lines 3x – 4y + 4 = 0 and 6x – 8y – 7 = 0 are tangents to a circle, then find the radius of the circle.


Answer:

Given that the lines are 3x - 4y + 4 = 0 and 6x - 8y - 7 = 0 are the tangents to the circle.



Let us find the slope of each line.


We know that the slope of the line ax + by + c = 0 is .


⇒ The slope of the line 3x - 4y + 4 = 0 is


⇒ The slope of the line 6x - 8y - 7 = 0 is


The slopes are equal. So, these are the tangents on either side of the diameter of the circle as shown in the figure.


The length of the diameter is the perpendicular distance between these two parallel lines.


We know that the distance between the two parallel lines ax + by + c1 = 0 and ax + by + c2 = 0 is given by


Here the parallel are 3x - 4y + 4 = 0 and 6x - 8y - 7 = 0(or )


Let ‘d’ and ‘r’ be the diameter and radius of the circle.








∴The radius of the circle is units.



Question 28.

Show that the point (x, y) given by and lies on a circle for all real values of t such that - 1 ≤ t ≤ 1, where a is any given real number.


Answer:

Given:




We need to prove that the point (x, y) lies on a circle for real values of t such that - 1≤t≤1, where a is any given real number.


Consider x2 + y2,








⇒ x2 + y2 = a2


The point (x,y) lies on a circle.


∴Thus proved.



Question 29.

The circle x2 + y2 – 2x – 2y + 1 = 0 is rolled along the positive direction of x - axis and makes one complete roll. Find its equation in new - position.


Answer:

Given equation of the circle is x2 + y2 - 2x - 2y + 1 = 0.



We know that the equation of the circle with centre (p,q) and having radius ‘r’ is given by:


⇒ (x - p)2 + (y - q)2 = r2 ..... - (1)


Now,


⇒ x2 + y2 - 2x - 2y + 1 = 0


⇒ (x2 - 2x + 1) + (y2 - 2y + 1) = 1


⇒ (x - 1)2 + (y - 1)2 = (1)2 ..... (2)


Comparing (2) with (1), we get


⇒ Centre = (1,1) and radius = 1


It is told that the circle is rolled along the positive direction of x - axis and makes one complete roll.


We know that the complete roll of a circle covers the distance 2r, where r is the radius of the circle.


The centre of the circle as moves 2r in the positive direction of xthe - axis.


Let the d be the distance moved by the centre on completion of one roll.



⇒ d = 2


The new position of the centre is (1 + d,1)


⇒ Centre = (1 + 2π, 1)


We have circle with centre (1 + 2π, 1) and having radius 1 units.


We know that the equation of the circle with centre (p, q) and having radius ‘r’ is given by:


⇒ (x - p)2 + (y - q)2 = r2


Now we substitute the corresponding values in the equation:


⇒ (x - (1 + 2π))2 + (y - 1)2 = 12


⇒ (x - 1 - 2π)2 + (y - 1)2 = 1


⇒ x2 - (2 + 4π)x + (1 + 2π)2 + y2 - 2y + 1 = 1


⇒ x2 + y2 - (2 + 4π)x - 2y - (1 + 2π)2 = 0


∴The equations of the circles is x2 + y2 - (2 + 4π)x - 2y - (1 + 2π)2 = 0.



Question 30.

One diameter of the circle circumscribing the rectangle ABCD is 4y = x + 7. If the coordinates of A and B are ( - 3, 4) and (5, 4) respectively, find the equation of the circle.


Answer:

Given that x - 4y + 7 = 0 is one of the diameters of the circle circumscribing the rectangle ABCD.



The coordinates of A and B are ( - 3,4) and (5,4).


Let us assume E be the mid - point of the line AB and ‘O’ be the centre of the circle.




- - - - - - (1)


Let us find the equation of the line AB.


We know that the equation of the line passing through the points (x1, y1) and (x2, y2) is given by




⇒ y = 4 ..... - (2)


From the figure, we can see that the line EO is perpendicular to the line AB.


So, the equation of the line EO is x = 1.


We find the centre as it is the point of intersection of the lines x - 4y + 7 = 0 and x = 1.


On solving this, we get the centre to be (1, 2).


We have a circle with centre (1, 2) and passing through the point (5,4).


We know that the radius of the circle is the distance between the centre and any point on the radius. So, we find the radius of the circle.


We know that the distance between the two points (x1,y1) and (x2,y2) is .


Let us assume r is the radius of the circle.





..... (2)


We know that the equation of the circle with centre (p, q) and having radius ‘r’ is given by:


⇒ (x - p)2 + (y - q)2 = r2


Now we substitute the corresponding values in the equation:



⇒ x2 - 2x + 1 + y2 - 4y + 4 = 13


⇒ x2 + y2 - 2x - 4y - 8 = 0.


∴The equation of the circle is x2 + y2 - 2x - 4y - 8 = 0.



Question 31.

If the line 2x – y + 1 = 0 touches the circle at the point (2, 5) and the centre of the circle lies on the line x + y – 9 = 0. Find the equation of the circle.


Answer:

Given that the line 2x - y + 1 = 0 touches the circle at the point (2,5) and the centre lies on the line x + y - 9 = 0.



We know that normal at any point on the circle passes through the centre of the circle.


We have tangent 2x - y + 1 = 0 at the point (2,5) for the circle.


We know that normal is perpendicular to the tangent and passes through the point of contact.


We know that product of slopes of two perpendicular lines is - 1.


Let us assume m be the slope of the normal.


We know that slope of a line ax + by + c = 0 is .


Slope of the line 2x - y + 1 = 0 is


⇒ m.(2) = - 1


- - - - - (1)


We know that the equation of a straight line passing through the point (x1, y1) and having slope ‘m’ is given by (y - y1) = m(x - x1).



⇒ 2y - 10 = - x + 2


⇒ x + 2y - 12 = 0 ..... (2)


We get the centre by solving for the intersecting point of the lines x + 2y - 12 = 0 and x + y - 9 = 0 as they both pass through the centre.


On solving these two lines we get,


⇒ Centre = (6, 3)


We have circle with centre (6, 3) and passing through the point (2, 5).


We know that radius of the circle is the distance between the centre and any point on the radius. So, we find the radius of the circle.


We know that the distance between the two points (x1,y1) and (x2,y2) is .


Let us assume r is the radius of the circle.





..... (2)


We know that the equation of the circle with centre (p, q) and having radius ‘r’ is given by:


⇒ (x - p)2 + (y - q)2 = r2


Now we substitute the corresponding values in the equation:



⇒ x2 - 12x + 36 + y2 - 6y + 9 = 20


⇒ x2 + y2 - 12x - 6y + 25 = 0.


∴The equation of the circle is x2 + y2 - 12x - 6y + 25 = 0.




Exercise 24.2
Question 1.

Find the coordinates of the centre radius of each of the following circle:

x2 + y2 + 6x - 8y - 24 = 0


Answer:

Given equation of the circle is x2 + y2 + 6x - 8y - 24 = 0 …… (1)



We know that for a circle x2 + y2 + 2ax + 2by + c = 0 ……(2)


⇒ Centre = (- a, - b)


⇒ Radius =


Comparing (1) with (2) we get,


⇒ Centre =


⇒ Centre = (- 3,4)


⇒ Radius =


⇒ Radius =


⇒ Radius = √49


⇒ Radius = 7


∴ The centre and radius of the circle is (- 3,4) and 7.



Question 2.

Find the coordinates of the centre radius of each of the following circle:

2x2 + 2y2 – 3x + 5y = 7


Answer:

Given equation of the circle is 2x2 + 2y2 - 3x + 5y = 7


..... (1)



We know that for a circle x2 + y2 + 2ax + 2by + c = 0 …… (2)


⇒ Centre = (- a, - b)


⇒ Radius =


Comparing (1) with (2) we get,


⇒ Centre =


⇒ Centre =


⇒ Radius =


⇒ Radius =


⇒ Radius =


⇒ Radius =


∴ The centre and radius of the circle is and .



Question 3.

Find the coordinates of the centre radius of each of the following circle:



Answer:

Given: the equation of the circle is



⇒ x2 + y2 + 2xcosθ + 2ysinθ - 8 = 0 ..... (1)


We know that for a circle


x2 + y2 + 2ax + 2by + c = 0 .... (2)


⇒ Centre = (- a, - b)


⇒ Radius =


Comparing (1) with (2) we get,


⇒ Centre =


⇒ Centre = (- cosθ, - sinθ)


⇒ Radius =


⇒ Radius =


⇒ Radius =


⇒ Radius = √9


⇒ Radius = 3


∴ The centre and radius of the circle is (- cosθ, - sinθ) and 3.



Question 4.

Find the coordinates of the centre radius of each of the following circle:

x2 + y2 – ax – by = 0


Answer:

Given:


equation of the circle is x2 + y2 - ax - by = 0 .... (1)



We know that for a circle


x2 + y2 + 2ax + 2by + c = 0 .... (2)


⇒ Centre = (- a, - b)


⇒ Radius =


Comparing (1) with (2) we get,


⇒ Centre =


⇒ Centre =


⇒ Radius =


⇒ Radius =


⇒ Radius =


⇒ Radius =


∴ The centre and radius of the circle is and .



Question 5.

Find the equation of the circle passing through the points :

(5, 7), (8, 1) and (1, 3)


Answer:

Given that we need to find the equation of the circle passing through the points (5,7), (8,1) and (1,3).



We know that the standard form of the equation of a circle is given by:


⇒ x2 + y2 + 2ax + 2by + c = 0 ..... (1)


Substituting (5,7) in (1), we get


⇒ 52 + 72 + 2a(5) + 2b(7) + c = 0


⇒ 25 + 49 + 10a + 14b + c = 0


⇒ 10a + 14b + c + 74 = 0 ..... (2)


Substituting (8,1) in (1), we get


⇒ 82 + 12 + 2a(8) + 2b(1) + c = 0


⇒ 64 + 1 + 16a + 2b + c = 0


⇒ 16a + 2b + c + 65 = 0 ..... (3)


Substituting (1,3) in (1), we get


⇒ 12 + 32 + 2a(1) + 2b(3) + c = 0


⇒ 1 + 9 + 2a + 6b + c = 0


⇒ 2a + 6b + c + 10 = 0 ..... (4)


Solving (2), (3), (4) we get


.


Substituting these values in (1), we get




⇒ 3x2 + 3y2 - 29x - 19y + 56 = 0


∴ The equation of the circle is 3x2 + 3y2 - 29x - 19y + 56 = 0.



Question 6.

Find the equation of the circle passing through the points :

(1, 2), (3, - 4) and (5, - 6)


Answer:

Given that we need to find the equation of the circle passing through the points (1,2), (3, - 4) and (5, - 6).



We know that the standard form of the equation of a circle is given by:


⇒ x2 + y2 + 2ax + 2by + c = 0 ..... (1)


Substituting (1,2) in (1), we get


⇒ 12 + 22 + 2a(1) + 2b(2) + c = 0


⇒ 1 + 4 + 2a + 4b + c = 0


⇒ 2a + 4b + c + 5 = 0 ..... (2)


Substituting (3, - 4) in (1), we get


⇒ 32 + (- 4)2 + 2a(3) + 2b(- 4) + c = 0


⇒ 9 + 16 + 6a - 8b + c = 0


⇒ 6a - 8b + c + 25 = 0 ..... (3)


Substituting (5, - 6) in (1), we get


⇒ 52 + (- 6)2 + 2a(5) + 2b(- 6) + c = 0


⇒ 25 + 36 + 10a - 12b + c = 0


⇒ 10a - 12b + c + 61 = 0 .....(4)


Solving (2), (3), (4) we get


⇒ a = - 11, b = - 2,c = 25.


Substituting these values in (1), we get


⇒ x2 + y2 + 2(- 11)x + 2(- 2) + 25 = 0


⇒ x2 + y2 - 22x - 4y + 25 = 0


∴ The equation of the circle is x2 + y2 - 22x - 4y + 25 = 0.



Question 7.

Find the equation of the circle passing through the points :

(5, - 8), (- 2, 9) and (2, 1)


Answer:

Given that we need to find the equation of the circle passing through the points (5, - 8), (- 2,9) and (2,1).



We know that the standard form of the equation of a circle is given by:


⇒ x2 + y2 + 2ax + 2by + c = 0 ..... (1)


Substituting (5, - 8) in (1), we get


⇒ 52 + (- 8)2 + 2a(5) + 2b(- 8) + c = 0


⇒ 25 + 64 + 10a - 16b + c = 0


⇒ 10a - 16b + c + 89 = 0 ..... (2)


Substituting (- 2,9) in (1), we get


⇒ (- 2)2 + 92 + 2a(- 2) + 2b(9) + c = 0


⇒ 4 + 81 - 4a + 18b + c = 0


⇒ - 4a + 18b + c + 85 = 0 ..... (3)


Substituting (2,1) in (1), we get


⇒ 22 + 12 + 2a(2) + 2b(1) + c = 0


⇒ 4 + 1 + 4a + 2b + c = 0


⇒ 4a + 2b + c + 5 = 0 ..... (4)


Solving (2), (3), (4) we get


⇒ a = 58, b = 24,c = - 285.


Substituting these values in (1), we get


⇒ x2 + y2 + 2(58)x + 2(24) - 285 = 0


⇒ x2 + y2 + 116x + 48y - 285 = 0


∴ The equation of the circle is x2 + y2 + 116x + 48y - 285 = 0.



Question 8.

Find the equation of the circle passing through the points :

(0, 0), (- 2, 1) and (- 3, 2)


Answer:

Given that we need to find the equation of the circle passing through the points (0,0), (- 2,1) and (- 3, 2).



We know that the standard form of the equation of a circle is given by:


⇒ x2 + y2 + 2ax + 2by + c = 0 ..... (1)


Substituting (0,0) in (1), we get


⇒ 02 + 02 + 2a(0) + 2b(0) + c = 0


⇒ 0 + 0 + 0a + 0b + c = 0


⇒ c = 0 ..... (2)


Substituting (- 2,1) in (1), we get


⇒ (- 2)2 + 12 + 2a(- 2) + 2b(1) + c = 0


⇒ 4 + 1 - 4a + 2b + c = 0


⇒ - 4a + 2b + c + 5 = 0 ..... (3)


Substituting (- 3,2) in (1), we get


⇒ (- 3)2 + 22 + 2a(- 3) + 2b(2) + c = 0


⇒ 9 + 4 - 6a + 4b + c = 0


⇒ - 6a + 4b + c + 13 = 0 ..... (4)


Solving (2), (3), (4) we get


.


Substituting these values in (1), we get



⇒ x2 + y2 - 3x - 11y = 0


∴ The equation of the circle is x2 + y2 - 3x - 11y = 0.



Question 9.

Find the equation of the circle which passes through (3, - 2), (- 2, 0) and has its centre on the line 2x – y = 3.


Answer:

Given that we need to find the equation of the circle which passes through (3, - 2), (- 2,0) and has its centre on the line 2x - y = 3. ..... (1)



We know that the standard form of the equation of the circle is given by:


⇒ x2 + y2 + 2ax + 2by + c = 0 .....(2)


Substituting centre (- a, - b) in (1) we get,


⇒ 2(- a) - (- b) = 3


⇒ - 2a + b = 3


⇒ 2a - b + 3 = 0 ......(3)


Substituting (3, - 2) in (2), we get


⇒ 32 + (- 2)2 + 2a(3) + 2b(- 2) + c = 0


⇒ 9 + 4 + 6a - 4b + c = 0


⇒ 6a - 4b + c + 13 = 0 ..... (4)


Substituting (- 2,0) in (2), we get


⇒ (- 2)2 + 02 + 2a(- 2) + 2b(0) + c = 0


⇒ 4 + 0 - 4a + c = 0


⇒ 4a - c - 4 = 0 ..... (5)


Solving (3), (4) and (5) we get,



Substituting these values in (2), we get



⇒ x2 + y2 + 3x + 12y + 2 = 0


∴ The equation of the circle is x2 + y2 + 3x + 12y + 2 = 0.



Question 10.

Find the equation of the circle which passes through the points (3, 7), (5, 5) and has its centre on line x – 4y = 1.


Answer:

Given that we need to find the equation of the circle which passes through (3,7), (5,5) and has its centre on the line x - 4y = 1. ..... (1)



We know that the standard form of the equation of the circle is given by:


⇒ x2 + y2 + 2ax + 2by + c = 0 .....(2)


Substituting centre (- a, - b) in (1) we get,


⇒ (- a) - 4(- b) = 1


⇒ - a + 4b = 1


⇒ a - 4b + 1 = 0 ......(3)


Substituting (3,7) in (2), we get


⇒ 32 + 72 + 2a(3) + 2b(7) + c = 0


⇒ 9 + 49 + 6a + 14b + c = 0


⇒ 6a + 14b + c + 58 = 0 ..... (4)


Substituting (5,5) in (2), we get


⇒ 52 + 52 + 2a(5) + 2b(5) + c = 0


⇒ 25 + 25 + 10a + 10b + c = 0


⇒ 10a + 10b + c + 50 = 0 ..... (5)


Solving (3), (4) and (5) we get,


⇒ a = 3,b = 1,c = - 90


Substituting these values in (2), we get


⇒ x2 + y2 + 2(3)x + 2(1)y - 90 = 0


⇒ x2 + y2 + 6x + 2y - 90 = 0


∴ The equation of the circle is x2 + y2 + 6x + 2y - 90 = 0.



Question 11.

Show that the points (3, - 2), (1, 0), (- 1, - 2) and (1, - 4) are con - cyclic.


Answer:

Given that we need to show the points A(3, - 2), B(1,0), C(- 1, - 2) and D(1, - 4) are con - cyclic.



The term con - cyclic means the points lie on the same circle.


Let us make a circle with any three points and check whether the fourth point lies on it or not.


Let us assume the circle passes through the points A, B, C.


We know that the standard form of the equation of the circle is given by:


⇒ x2 + y2 + 2ax + 2by + c = 0 ..... (1)


Substituting A(3, - 2) in (1), we get,


⇒ 32 + (- 2)2 + 2a(3) + 2b(- 2) + c = 0


⇒ 9 + 4 + 6a - 4b + c = 0


⇒ 6a - 4b + c + 13 = 0 ..... (2)


Substituting B(1,0) in (1), we get,


⇒ 12 + 02 + 2a(1) + 2b(0) + c = 0


⇒ 1 + 2a + c = 0 ......- (3)


Substituting C(- 1, - 2) in (1), we get,


⇒ (- 1)2 + (- 2)2 + 2a(- 1) + 2b(- 2) + c = 0


⇒ 1 + 4 - 2a - 4b + c = 0


⇒ 5 - 2a - 4b + c = 0


⇒ 2a + 4b - c - 5 = 0 ..... (4)


On solving (2), (3) and (4) we get,


⇒ a = - 1, b = 2 and c = 1


Substituting these values in (1), we get


⇒ x2 + y2 + 2(- 1)x + 2(2)y + 1 = 0


⇒ x2 + y2 - 2x + 4y + 1 = 0 ..... (5)


Substituting D(1, - 4) in eq(5) we get,


⇒ 12 + (- 4)2 - 2(1) + 4(- 4) + 1


⇒ 1 + 16 - 2 - 16 + 1


⇒ 0


∴ The points (3, - 2), (1,0), (- 1, - 2), (1, - 4) are con - cyclic.



Question 12.

Show that the points (5, 5), (6, 4), (- 2, 4) and (7, 1) all lie on a circle, and find its equation, centre, and radius.


Answer:

Given that we need to show the points A(5,5), B(6,4), C(- 2,4) and D(7,1) lie on a circle.



Let us make a circle with any three points and check whether the fourth point lies on it or not.


Let us assume the circle passes through the points A, B, C.


We know that the standard form of the equation of the circle is given by:


⇒ x2 + y2 + 2ax + 2by + c = 0 ..... (1)


Substituting A(5,5) in (1), we get,


⇒ 52 + 52 + 2a(5) + 2b(5) + c = 0


⇒ 25 + 25 + 10a + 10b + c = 0


⇒ 10a + 10b + c + 50 = 0 ..... (2)


Substituting B(6,4) in (1), we get,


⇒ 62 + 42 + 2a(6) + 2b(4) + c = 0


⇒ 36 + 16 + 12a + 8b + c = 0


⇒ 12a + 8b + c + 52 = 0 .....(3)


Substituting C(- 2,4) in (1), we get,


⇒ (- 2)2 + 42 + 2a(- 2) + 2b(4) + c = 0


⇒ 4 + 16 - 4a + 8b + c = 0


⇒ 20 - 4a + 8b + c = 0


⇒ 4a - 8b - c - 20 = 0 .....(4)


On solving (2), (3) and (4) we get,


⇒ a = - 2, b = - 1 and c = - 20


Substituting these values in (1), we get


⇒ x2 + y2 + 2(- 2)x + 2(- 1)y - 20 = 0


⇒ x2 + y2 - 4x - 2y - 20 = 0 ..... (5)


Substituting D(7,1) in eq(5) we get,


⇒ 72 + 12 - 4(7) - 2(1) - 20


⇒ 49 + 1 - 28 - 2 - 20


⇒ 0


∴ The points (3, - 2), (1,0), (- 1, - 2), (1, - 4) lie on a circle.


We know that for a circle x2 + y2 + 2ax + 2by + c = 0,


⇒ Centre = (- a, - b)


⇒ Radius =


Comparing (5) with (1), we get


⇒ Centre =


⇒ Centre = (2,1)


⇒ Radius =


⇒ Radius =


⇒ Radius = 5.


∴ The centre and radius of the circle is (2, 1) and 5.



Question 13.

Find the equation of the circle which circumscribes the triangle formed by the lines:

x + y + 3 = 0, x - y + 1 = 0 and x = 3


Answer:

Given that we need to find the equation of the circle formed by the lines:



⇒ x + y + 3 = 0


⇒ x - y + 1 = 0


⇒ x = 3


On solving these lines we get the intersection points A(- 2, - 1), B(3,4), C(3, - 6)


We know that the standard form of the equation of a circle is given by:


⇒ x2 + y2 + 2ax + 2by + c = 0 ..... (1)


Substituting (- 2, - 1) in (1), we get


⇒ (- 2)2 + (- 1)2 + 2a(- 2) + 2b(- 1) + c = 0


⇒ 4 + 1 - 4a - 2b + c = 0


⇒ 5 - 4a - 2b + c = 0


⇒ 4a + 2b - c - 5 = 0 ..... (2)


Substituting (3,4) in (1), we get


⇒ 32 + 42 + 2a(3) + 2b(4) + c = 0


⇒ 9 + 16 + 6a + 8b + c = 0


⇒ 6a + 8b + c + 25 = 0 ..... (3)


Substituting (3, - 6) in (1), we get


⇒ 32 + (- 6)2 + 2a(3) + 2b(- 6) + c = 0


⇒ 9 + 36 + 6a - 12b + c = 0


⇒ 6a - 12b + c + 45 = 0 ..... (4)


Solving (2), (3), (4) we get


⇒ a = - 3, b = 1,c = - 15.


Substituting these values in (1), we get


⇒ x2 + y2 + 2(- 3)x + 2(1)y - 15 = 0


⇒ x2 + y2 - 6x + 2y - 15 = 0


∴ The equation of the circle is x2 + y2 - 6x + 2y - 15 = 0.



Question 14.

Find the equation of the circle which circumscribes the triangle formed by the lines:

2x + y - 3 = 0, x + y - 1 = 0 and 3x + 2y - 5 = 0


Answer:

Given that we need to find the equation of the circle formed by the lines:



⇒ 2x + y - 3 = 0


⇒ x + y - 1 = 0


⇒ 3x + 2y - 5 = 0


On solving these lines we get the intersection points A(2, - 1), B(3, - 2), C(1,1)


We know that the standard form of the equation of a circle is given by:


⇒ x2 + y2 + 2ax + 2by + c = 0 .....(1)


Substituting (2, - 1) in (1), we get


⇒ 22 + (- 1)2 + 2a(2) + 2b(- 1) + c = 0


⇒ 4 + 1 + 4a - 2b + c = 0


⇒ 4a - 2b + c + 5 = 0 ..... (2)


Substituting (3, - 2) in (1), we get


⇒ 32 + (- 2)2 + 2a(3) + 2b(- 2) + c = 0


⇒ 9 + 4 + 6a - 4b + c = 0


⇒ 6a - 4b + c + 13 = 0 ..... (3)


Substituting (1,1) in (1), we get


⇒ 12 + 12 + 2a(1) + 2b(1) + c = 0


⇒ 1 + 1 + 2a + 2b + c = 0


⇒ 2a + 2b + c + 2 = 0 ..... (4)


Solving (2), (3), (4) we get


.


Substituting these values in (1), we get



⇒ x2 + y2 - 13x - 5y + 16 = 0


∴ The equation of the circle is x2 + y2 - 13x - 5y + 16 = 0.



Question 15.

Find the equation of the circle which circumscribes the triangle formed by the lines:

x + y = 2, 3x - 4y = 6 and x - y = 0


Answer:

Given that we need to find the equation of the circle formed by the lines:



⇒ x + y = 2


⇒ 3x - 4y = 6


⇒ x - y = 0


On solving these lines we get the intersection points A(2,0), B(- 6, - 6), C(1,1)


We know that the standard form of the equation of a circle is given by:


⇒ x2 + y2 + 2ax + 2by + c = 0 .....(1)


Substituting (2,0) in (1), we get


⇒ 22 + 02 + 2a(2) + 2b(0) + c = 0


⇒ 4 + 4a + c = 0


⇒ 4a + c + 4 = 0 ..... (2)


Substituting (- 6, - 6) in (1), we get


⇒ (- 6)2 + (- 6)2 + 2a(- 6) + 2b(- 6) + c = 0


⇒ 36 + 36 - 12a - 12b + c = 0


⇒ 12a + 12b - c - 72 = 0 ..... (3)


Substituting (1,1) in (1), we get


⇒ 12 + 12 + 2a(1) + 2b(1) + c = 0


⇒ 1 + 1 + 2a + 2b + c = 0


⇒ 2a + 2b + c + 2 = 0 ..... (4)


Solving (2), (3), (4) we get


⇒ a = 2, b = 3,c = - 12.


Substituting these values in (1), we get


⇒ x2 + y2 + 2(2)x + 2(3)y - 12 = 0


⇒ x2 + y2 + 4x + 6y - 12 = 0


∴ The equation of the circle is x2 + y2 + 4x + 6y - 12 = 0.



Question 16.

Find the equation of the circle which circumscribes the triangle formed by the lines:

y = x + 2, 3y = 4x and 2y = 3x


Answer:

Given that we need to find the equation of the circle formed by the lines:



⇒ y = x + 2


⇒ 3y = 4x


⇒ 2y = 3x


On solving these lines we get the intersection points A(6,8), B(0,0), C(4,6)


We know that the standard form of the equation of a circle is given by:


⇒ x2 + y2 + 2ax + 2by + c = 0 .....(1)


Substituting (6,8) in (1), we get


⇒ 62 + 82 + 2a(6) + 2b(8) + c = 0


⇒ 36 + 64 + 12a + 16b + c = 0


⇒ 12a + 16b + c + 100 = 0 ......(2)


Substituting (0,0) in (1), we get


⇒ 02 + 02 + 2a(0) + 2b(0) + c = 0


⇒ 0 + 0 + 0a + 0b + c = 0


⇒ c = 0 .....(3)


Substituting (4,6) in (1), we get


⇒ 42 + 62 + 2a(4) + 2b(6) + c = 0


⇒ 16 + 36 + 8a + 12b + c = 0


⇒ 8a + 12b + c + 52 = 0 ..... (4)


Solving (2), (3), (4) we get


⇒ a = - 23, b = 11,c = 0.


Substituting these values in (1), we get


⇒ x2 + y2 + 2(- 23)x + 2(11)y + 0 = 0


⇒ x2 + y2 - 46x + 22y = 0


∴ The equation of the circle is x2 + y2 - 46x + 22y = 0.



Question 17.

Prove that the centres of the three circles x2 + y2 – 4x – 6y – 12 = 0, x2 + y2 + 2x + 4y – 10 = 0 and x2 + y2 – 10x – 16y – 1 = 0 are collinear.


Answer:

Given circles are:



(i) x2 + y2 - 4x - 6y - 12 = 0


(ii) x2 + y2 + 2x + 4y - 10 = 0


(iii) x2 + y2 - 10x - 16y - 1 = 0


We know that for a circle x2 + y2 + 2ax + 2by + c = 0 - - (1)


⇒ Centre = (- a, - b)


⇒ Radius =


Comparing (i) with (1) we get,


⇒ Centre(A) =


⇒ A = (2,3)


Comparing (ii) with (1) we get,


⇒ Centre(B) =


⇒ B = (- 1, - 2)


Comparing (iii) with (1) we get,


⇒ Centre(C) =


⇒ C = (5,8)


We need to show that A, B, and C are collinear.


We find the line passing through any two points and check whether the third point is present on it or not, by substituting the point in the line.


Let us assume the line passes through the points A, B.


We know that the equation of the line passing through the points (x1,y1) and (x2,y2) is .




⇒ 3(y - 3) = 5(x - 2)


⇒ 3y - 9 = 5x - 10


⇒ 5x - 3y - 1 = 0 ..... (2)


Substituting point C(5,8) in (2), we get


⇒ 5(5) - 3(8) - 1


⇒ 25 - 24 - 1


⇒ 0


∴ The centres of the circle are collinear.



Question 18.

Prove that the radii of the circles x2 + y2 = 1, x2 + y2 – 2x – 6y – 6 = 0 and x2 + y2 – 4x – 12y – 9 = 0 are in A. P.


Answer:

Given circles are:



(i) x2 + y2 - 1 = 0


(ii) x2 + y2 - 2x - 6y - 6 = 0


(iii) x2 + y2 - 4x - 12y - 9 = 0


We know that for a circle x2 + y2 + 2ax + 2by + c = 0 - - (1)


⇒ Centre = (- a, - b)


⇒ Radius =


Comparing (i) with (1) we get,


⇒ Radius (r1) =


⇒ r1 = 1


Comparing (ii) with (1) we get,


⇒ Radius (r2) =



⇒ r2 = √16


⇒ r2 = 4


Comparing (iii) with (1) we get,


⇒ Radius (r3) =



⇒ r3 = √49


⇒ r3 = 7


We need to show r1,r2,r3 are in A.P.


We know that for three numbers a, b, c to be in A.P. The condition to be satisfied is:






⇒ b = 4


⇒ b = r2


∴ The radii r1,r2,r3 are in A.P.



Question 19.

Find the equation of the circle which passes through the origin and cuts off chords of lengths 4 and 6 on the positive side of the x - axis and y - axis respectively.


Answer:

Given that we need to find the equation of the circle which passes through the origin and cuts off chords of lengths 4 and 6 on the positive side of the x - axis and y - axis.



Since the circle passes through origin O. So, the points on x and y - axis which are intersected by the circle are A(4, 0) and B(0, 6).


The mid - point of O(0,0) and A(4,0) is C(2,0) and that of O(0,0) and B(0,6) is D(0,3).


Let us assume that P is the centre of the circle.


From the figure, we can see that the line PC is perpendicular bisector of the chord OA and line PD is perpendicular bisector of the chord OB.


We get the centre of the circle to be (2,3).


We have a circle with centre (2,3) and passing through the point (0,0).


We know that the radius of the circle is the distance between the centre and any point on the radius. So, we find the radius of the circle.


We know that the distance between the two points (x1,y1) and (x2,y2) is .


Let us assume r is the radius of the circle.





⇒ r = √13 ..... (1)


We know that the equation of the circle with centre (p, q) and having radius ‘r’ is given by:


⇒ (x - p)2 + (y - q)2 = r2


Now we substitute the corresponding values in the equation:



⇒ x2 - 4x + 4 + y2 - 6y + 9 = 13


⇒ x2 + y2 - 4x - 6y = 0.


∴ The equation of the circle is x2 + y2 - 4x - 6y = 0.



Question 20.

Find the equation of the circle concentric with the circle x2 + y2 – 6x + 12y + 15 = 0 and double of its area.


Answer:

Given that we need to find the equation of the circle which is concentric with x2 + y2 - 6x + 12y + 15 = 0 and double its area.



We know that concentric circles will have the same centre.


Let us assume the concentric circle be x2 + y2 - 6x + 12y + c = 0. .....(ii)


We know that for a circle x2 + y2 + 2ax + 2by + c = 0 - - (1)


⇒ Centre = (- a, - b)


⇒ Radius =


Let us assume the radius of the first circle is r1.





Given that the concentric circle’s area is double the first circle’s area.


Let us assume the radius of the concentric circle be r2.


We know that the area of the circle is πr2


Now,


⇒ Area of concentric circle = 2 × area of the first circle




⇒ r22 = 60


⇒ r2 = √60


Comparing with (ii) with (1), we get



⇒ 9 + 36 - c = 60


⇒ c = - 15


Substituting c value in (ii), we get


⇒ x2 + y2 - 6x + 12y - 15 = 0


∴ The equation of the concentric circle is x2 + y2 - 6x + 12y - 15 = 0.



Question 21.

Find the equation to the circle which passes through the points (1, 1)(2, 2) and whose radius is 1. Show that there are two such circles.


Answer:

Given that we need to find the equation of the circle which passes through the points (1,1), (2,2) and having radius 1.



Let us assume the equation of the circle is:


⇒ x2 + y2 + 2ax + 2by + c = 0 .....(1)


We know that the radius of the circle is



⇒ a2 + b2 - c = 1 .....(2)


Substituting the point (1,1) in (1) we get,


⇒ 12 + 12 + 2a(1) + 2b(1) + c = 0


⇒ 1 + 1 + 2a + 2b + c = 0


⇒ 2a + 2b + c + 2 = 0


..... (3)


Substituting the point (2,2) in (1) we get,


⇒ 22 + 22 + 2a(2) + 2b(2) + c = 0


⇒ 4 + 4 + 4a + 4b + c = 0


⇒ 4a + 4b + c + 8 = 0


..... (4)


Subtracting (3) from (4), we get



⇒ c = 4 ..... (5)


Substituting (5) in (3) we get



⇒ a + b + 2 = - 1


⇒ a + b = - 3


⇒ a = - 3 - b ......(6)


Substituting (6) in (2) we get,


⇒ (- 3 - b)2 + b2 - 4 = 1


⇒ 9 + b2 + 6b + b2 - 5 = 0


⇒ 2b2 + 6b + 4 = 0


⇒ b2 + 3b + 2 = 0


⇒ b2 + 2b + b + 2 = 0


⇒ b(b + 2) + 1(b + 2) = 0


⇒ (b + 1)(b + 2) = 0


⇒ b + 1 = 0 (or) b + 2 = 0


⇒ b = - 1 (or) b = - 2


For b = - 1, substituting in (6)


⇒ a = - 3 - (- 1)


⇒ a = - 2


For b = - 2, substituting in (6)


⇒ a = - 3 - (- 2)


⇒ a = - 1


Now for a = - 2, b = - 1 and c = 4, the equation of the circle is x2 + y2 - 4x - 2x + 4 = 0


For a = - 1, b = - 2 and c = 4, the equation of the circle is x2 + y2 - 2x - 4y + 4 = 0


∴ The equations of the circles are x2 + y2 - 4x - 2y + 4 = 0 and x2 + y2 - 2x - 4y + 4 = 0.



Question 22.

Find the equation of the circle concentric with x2 + y2 – 4x – 6y – 3 = 0 and which touches the y - axis.


Answer:

Given that we need to find the equation of the circle which is concentric with x2 + y2 - 4x - 6y - 3 = 0 which touches the y - axis.



We know that the concentric circles will have the same centre.


Let us assume the equation of the concentric circle be x2 + y2 - 4x - 6y + c = 0. ......- (1)


We know that the value of x is 0 on the y - axis.


Substituting x = 0 in (1), we get


⇒ 02 + y2 - 4(0) - 6y + c = 0


⇒ y2 - 6y + c = 0 ......- (2)


We need to get only similar roots on solving the quadratic equation (2), since the circle touches y - axis at only one point.


We know that for a quadratic equation ax2 + bx + c = 0, to have similar roots the condition need to be satisfied is:


⇒ b2 - 4ac = 0


From (2),


⇒ (- 6)2 - 4(1)(c) = 0


⇒ 36 - 4c = 0


⇒ 4c = 36



⇒ c = 9 ..... (3)


Substituting (3) in (1), we get


⇒ x2 + y2 - 4x - 6y + 9 = 0


∴ The equation of the concentric circle which touches y - axis is x2 + y2 - 4x - 6y + 9 = 0.



Question 23.

If a circle passes through the point (0, 0), (a, 0), (0, b), then find the coordinates of its centre.


Answer:

Given that the circle passes through the points O(0,0), A(a,0) and B(0,b).



Let us first find the length of the sides of the triangle formed by the points OAB.


We know that distance between two points (x1,y1) and (x2,y2) is .




⇒ OA = |a| .....(1)




⇒ OB = |b| .....(2)



..... (3)


Now consider OA2 + OB2,


⇒ OA2 + OB2 = (|a|)2 + (|b|)2


⇒ OA2 + OB2 = a2 + b2



⇒ OA2 + OB2 = AB2


We got ABC is a right angled triangle with AB as the hypotenuse.


We know that the circumcentre of a right-angled triangle is the midpoint of the hypotenuse.


⇒ Centre =


⇒ Centre =


∴ The coordinates of the centre is .



Question 24.

Find the equation of the circle which passes through the points (2, 3) and (4, 5) and the centre lies on the straight line y – 4x + 3 = 0.


Answer:

Given that we need to find the equation of the circle which passes through (2,3), (4,5) and has its centre on the line y - 4x + 3 = 0. ..... (1)



We know that the standard form of the equation of the circle is given by:


⇒ x2 + y2 + 2ax + 2by + c = 0 .....(2)


Substituting centre (- a, - b) in (1) we get,


⇒ - 4(- a) + (- b) + 3 = 0


⇒ 4a - b + 3 = 0 ......(3)


Substituting (2,3) in (2), we get


⇒ 22 + 32 + 2a(2) + 2b(3) + c = 0


⇒ 4 + 9 + 4a + 6b + c = 0


⇒ 4a + 6b + c + 13 = 0 ..... (4)


Substituting (4,5) in (2), we get


⇒ 42 + 52 + 2a(4) + 2b(5) + c = 0


⇒ 16 + 25 + 8a + 10b + c = 0


⇒ 8a + 10b + c + 41 = 0 ..... (5)


Solving (3), (4) and (5) we get,


⇒ a = - 2,b = - 5,c = 25


Substituting these values in (2), we get


⇒ x2 + y2 + 2(- 2)x + 2(- 5)y + 25 = 0


⇒ x2 + y2 - 4x - 10y + 25 = 0


∴ The equation of the circle is x2 + y2 - 4x - 10y + 25 = 0.




Exercise 24.3
Question 1.

Find the equation of the circle, the end points of whose diameter are (2, - 3) and (- 2, 4). Find its centre and radius.


Answer:

Given that we need to find the equation of the circle whose end points of a diameter are (2, - 3) and (- 2,4). The figure is given below:



We know that centre is the midpoint of the diameter.


⇒ Centre(C) =



We have a circle with centre and passing through the point (2, - 3).


We know that the radius of the circle is the distance between the centre and any point on the radius. So, we find the radius of the circle.


We know that the distance between the two points (x1,y1) and (x2,y2) is .


Let us assume r is the radius of the circle.







We know that the equation of the circle with centre (p, q) and having radius ‘r’ is given by:


⇒ (x - p)2 + (y - q)2 = r2


Now we substitute the corresponding values in the equation:




⇒ 4x2 + 4y2 - 4y + 1 = 65


∴The equation of the circle is 4x2 + 4y2 - 4y - 64 = 0 or x2 + y2 - y - 16 = 0


Question 2.

Find the equation of the circle the end points of whose diameter are the centres of the circles x2 + y2 + 6x – 14y – 1 = 0 and x2 + y2 – 4x + 10y – 2 = 0.


Answer:

Given that we need to find the equation of the circle whose end points of a diameter are the centres of the circles



x2 + y2 + 6x - 14y - 1 = 0 .... - (i) and


x2 + y2 - 4x + 10y - 2 = 0 ...... - (ii)


Let us assume A and B are the centres of the 1st and 2nd circle.


We know that for a circle x2 + y2 + 2ax + 2by + c = 0 ...... - (1)


⇒ Centre = (- a, - b)


⇒ Radius =


Comparing (i) with (1) we get,


⇒ Centre(A) =


⇒ A = (- 3,7)


Comparing (ii) with (1) we get,


⇒ Centre(B) =


⇒ B = (2, - 5)


We know that the centre is the mid - point of the diameter.


⇒ Centre(C) =



We have a circle with centre and passing through the point (2, - 5).


We know that the radius of the circle is the distance between the centre and any point on the radius. So, we find the radius of the circle.


We know that the distance between the two points (x1,y1) and (x2,y2) is .


Let us assume r is the radius of the circle.







We know that the equation of the circle with centre (p, q) and having radius ‘r’ is given by:


⇒ (x - p)2 + (y - q)2 = r2


Now we substitute the corresponding values in the equation:




⇒ x2 + y2 + x - 2y - 41 = 0


∴The equation of the circle is x2 + y2 + x - 2y - 41 = 0.



Question 3.

The sides of a squares are x = 6, x = 9, y = 3 and y = 6. Find the equation of a circle drawn on the diagonal of the square as its diameter.


Answer:

Given that we need to find the equation of the circle with the diagonal of a square as diameter.



It is also told that x = 6, x = 9, y = 3, and y = 6 are the sides of a square.


Let us assume A,B,C,D are the vertices of the square. On solving the lines, we get the vertices as:


⇒ A = (6,3)


⇒ B = (9,3)


⇒ C = (9,6)


⇒ D = (6,6)


Since the diagonal of the square is the diameter of the circle, the circle circumscribes the square.


So, taking any diagonal as diameter gives the same equation of the circle.


Let us assume diagonal BD as the diameter.


We know that the centre is the mid - point of the diameter.


⇒ Centre(O) =



We have a circle with centre and passing through the point (6,3).


We know that the radius of the circle is the distance between the centre and any point on the radius. So, we find the radius of the circle.


We know that the distance between the two points (x1,y1) and (x2,y2) is .


Let us assume r is the radius of the circle.






We know that the equation of the circle with centre (p, q) and having radius ‘r’ is given by:


⇒ (x - p)2 + (y - q)2 = r2


Now we substitute the corresponding values in the equation:




⇒ x2 + y2 - 15x - 9y + 72 = 0


∴The equation of the circle is x2 + y2 - 15x - 9y + 72 = 0.



Question 4.

Find the equation of the circle circumscribing the rectangle whose sides are x – 3y = 4, 3x + y = 22, x – 3y = 14 and 3x + y = 62.


Answer:

Given that we need to find the equation of the circle which circumscribes the rectangle.



It is also told that x - 3y = 4, 3x + y = 22, x - 3y = 14 and 3x + y = 62 are the sides of a rectangle.


Let us assume A,B,C,D are the vertices of the rectangle. On solving the lines, we get the vertices as:


⇒ A = (7,1)


⇒ B = (8, - 2)


⇒ C = (20,2)


⇒ D = (19,5)


Since the circle circumscribes the rectangle, the diagonal of the rectangle will be the diameter of the circle.


So, taking any diagonal as diameter gives the same equation of the circle.


Let us assume diagonal AC as the diameter.


We know that the centre is the mid - point of the diameter.


⇒ Centre(C) =



We have a circle with a centre and passing through the point (7,1).


We know that the radius of the circle is the distance between the centre and any point on the radius. So, we find the radius of the circle.


We know that the distance between the two points (x1,y1) and (x2,y2) is .


Let us assume r is the radius of the circle.







We know that the equation of the circle with centre (p, q) and having radius ‘r’ is given by:


⇒ (x - p)2 + (y - q)2 = r2


Now we substitute the corresponding values in the equation:




⇒ x2 + y2 - 27x - 3y + 142 = 0


∴The equation of the circle is x2 + y2 - 27x - 3y + 142 = 0.



Question 5.

Find the equation of the circle passing through the origin and the points where the line 3x + 4y = 12 meets the axes of coordinates.


Answer:

Given that we need to find the equation of the circle passing through the origin and the points where the line 3x + 4y = 12 meets the axes of co - ordinates.



Let us first find the points at which the line meets the axes.


The value of x is 0 on meeting the y - axis. So,


⇒ 3(0) + 4y = 12


⇒ 4y = 12


⇒ y = 3


The point is A(0,3)


The value of y is 0 on meeting the x - axis. So,


⇒ 3x + 4(0) = 12


⇒ 3x = 12


⇒ x = 4


The point is B(4,0)


We have the circle passing through the points O(0,0), A(0,3) and B(4,0).


We know that the standard form of the equation of the circle is given by:


⇒ x2 + y2 + 2ax + 2by + c = 0 ..... (1)


Substituting O(0,0) in (1), we get,


⇒ 02 + 02 + 2a(0) + 2b(0) + c = 0


⇒ c = 0 ..... (2)


Substituting A(0,3) in (1), we get,


⇒ 02 + 32 + 2a(0) + 2b(3) + c = 0


⇒ 9 + 6b + c = 0


⇒ 6b + c + 9 = 0 ..... (3)


Substituting B(4,0) in (1), we get,


⇒ 42 + 02 + 2a(4) + 2b(0) + c = 0


⇒ 16 + 8a + c = 0


⇒ 8a + c + 16 = 0 ..... (4)


On solving (2), (3) and (4) we get,



Substituting these values in (1), we get



⇒ x2 + y2 - 4x - 3y = 0


∴The equation of the circle is x2 + y2 - 4x - 3y = 0.



Question 6.

Find the equation of the circle which passes through the origin and cuts off intercepts a and b respectively from x and y - axes.


Answer:

Given that we need to find the equation of the circle passing through the origin and cuts off intercepts a and b from x and y - axes.



Since the circle has intercept a from x - axis the circle must pass through (a,0) and (- a,0) as it already passes through the origin.


Since the circle has intercept b from x - axis, the circle must pass through (0,b) and (0, - b) as it already passes through the origin.


Let us assume the circle passing through the points O(0,0), A(a,0) and B(0,b).


We know that the standard form of the equation of the circle is given by:


⇒ x2 + y2 + 2fx + 2gy + c = 0 ..... (1)


Substituting O(0,0) in (1), we get,


⇒ 02 + 02 + 2f(0) + 2g(0) + c = 0


⇒ c = 0 ..... (2)


Substituting A(a,0) in (1), we get,


⇒ a2 + 02 + 2f(a) + 2g(0) + c = 0


⇒ a2 + 2fa + c = 0 ..... (3)


Substituting B(0,b) in (1), we get,


⇒ 02 + b2 + 2f(0) + 2g(b) + c = 0


⇒ b2 + 2gb + c = 0 ..... (4)


On solving (2), (3) and (4) we get,



Substituting these values in (1), we get



⇒ x2 + y2 - ax - by = 0


Similarly, we get the equation x2 + y2 + ax + by = 0 for the circle passing through the points (0,0), (- a,0), (0, - b).


∴The equations of the circles are x2 + y2±ax±by = 0.



Question 7.

Find the equation of the circle whose diameter is the line segment joining (- 4, 3) and (12, - 1). Find also the intercept made by it on the y-axis.


Answer:

Given that we need to find the equation of the circle whose end points of a diameter are (- 4,3) and (12, - 1).



We know that the centre is the mid - point of the diameter.


⇒ Centre(C) =


⇒ C = (4,1)


We have a circle with centre (4,1) and passing through the point (- 4,3).


We know that the radius of the circle is the distance between the centre and any point on the radius. So, we find the radius of the circle.


We know that the distance between the two points (x1,y1) and (x2,y2) is .


Let us assume r is the radius of the circle.






We know that the equation of the circle with centre (p, q) and having radius ‘r’ is given by:


⇒ (x - p)2 + (y - q)2 = r2


Now we substitute the corresponding values in the equation:



⇒ x2 - 8x + 16 + y2 - 2y + 1 = 68


⇒ x2 + y2 - 8x - 2y - 51 = 0 ..... (1)


To find the y - intercept, we need the find the points at which the circle intersects the y - axis.


We know that x = 0 on y - axis. Substituting x = 0 in (1) we get


⇒ 02 + y2 - 8(0) - 2y - 51 = 0


⇒ y2 - 2y - 51 = 0




⇒ y = 1 ± 2√13


∴The y - intercepts are 1 ± 2√13.



Question 8.

The abscissae of the two points A and B are the roots of the equation x2 + 2ax – b2 = 0 and their ordinates are the roots of the equation x2 + 2px – q2 = 0. Find the equation of the circle with AB as diameter. Also, find its radius.


Answer:

Given that points, we need to find the equation of the circle whose ends of the diameter are A and B.


It is also told that the abscissae of two points A and B are the roots of x2 + 2ax - b2 = 0 and ordinates are the roots of x2 + 2px - q2 = 0.


Let us first find the roots of each quadratic equation.


For x2 + 2ax - b2 = 0




..... (1)


For x2 + 2px - q2 = 0




..... (2)


From (1) and (2) we get,




We know that the centre is the mid - point of the diameter.


⇒ Centre(C) =


⇒ C = (- a, - p)


We have a circle with centre (- a, - p) and passing through the point .


We know that the radius of the circle is the distance between the centre and any point on the radius. So, we find the radius of the circle.


We know that the distance between the two points (x1,y1) and (x2,y2) is .


Let us assume r is the radius of the circle.





We know that the equation of the circle with centre (p, q) and having radius ‘r’ is given by:


⇒ (x - p)2 + (y - q)2 = r2


Now we substitute the corresponding values in the equation:



⇒ x2 + 2ax + a2 + y2 + 2py + p2 = a2 + b2 + p2 + q2


⇒ x2 + y2 + 2ax + 2py - b2 - q2 = 0


∴The equation of the circle is x2 + y2 + 2ax + 2py - b2 - q2 = 0.



Question 9.

ABCD is a square whose side is a; taking AB and AD as axes, prove that the equation of the circle circumscribing the square is x2 + y2 – a(x + y) = 0.


Answer:


Given that we need to find the equation of the circle which circumscribes the square ABCD of side a.


It is also told that AB and AD are assumed as x and y - axes.


Assuming A as origin, We get the points B(a,0) and D(0,a).


So, we need to find the circle which is passing through the points A(0,0), B(a,0) and D(0,a).


We know that the standard form of the equation of the circle is given by:


⇒ x2 + y2 + 2fx + 2gy + c = 0 ..... (1)


Substituting A(0,0) in (1), we get,


⇒ 02 + 02 + 2f(0) + 2g(0) + c = 0


⇒ c = 0 ..... (2)


Substituting B(a,0) in (1), we get,


⇒ a2 + 02 + 2f(a) + 2g(0) + c = 0


⇒ a2 + 2fa + c = 0 ..... (3)


Substituting D(0,a) in (1), we get,


⇒ 02 + a2 + 2f(0) + 2g(a) + c = 0


⇒ a2 + 2ga + c = 0 ..... (4)


On solving (2), (3) and (4) we get,



Substituting these values in (1), we get



⇒ x2 + y2 - ax - ay = 0


⇒ x2 + y2 - a(x + y) = 0


∴Thus proved.



Question 10.

The line 2x – y + 6 = 0 meets the circle x2 + y2 – 2y – 9 = 0 at A and B. Find the equation of the circle on AB as diameter.


Answer:

Given that we need to find the equation of the circle whose diameter is AB.



It is also told that the points A and B are the intersection points when the line 2x - y + 6 = 0 meets the circle


x2 + y2 - 2y - 9 = 0. ..... - (1)


Let us first find the points A and B.


From the equation of the line:


⇒ 2x - y + 6 = 0


⇒ y = 2x + 6 ..... (2)


Substituting (2) in (1), we get


⇒ x2 + (2x + 6)2 - 2(2x + 6) - 9 = 0


⇒ x2 + 4x2 + 24x + 36 - 4x - 12 - 9 = 0


⇒ 5x2 + 20x + 15 = 0


⇒ 5x2 + 5x + 15x + 15 = 0


⇒ 5x(x + 1) + 15(x + 1) = 0


⇒ (5x + 15)(x + 1) = 0


⇒ 5x + 15 = 0 or x + 1 = 0


⇒ 5x = - 15 or x = - 1


⇒ x = - 3 or x = - 1


For x = - 3, from (2)


⇒ y = 2(- 3) + 6


⇒ y = - 6 + 6


⇒ y = 0


For x = - 1, from (2)


⇒ y = 2(- 1) + 6


⇒ y = - 2 + 6


⇒ y = 4


The points are A(- 3,0) and B(- 1,4)


We know that centre is the mid - point of the diameter.


⇒ Centre(C) =


⇒ C = (- 2,2)


We have circle with centre (- 2,2) and passing through the point (- 1,4).


We know that the radius of the circle is the distance between the centre and any point on the radius. So, we find the radius of the circle.


We know that the distance between the two points (x1,y1) and (x2,y2) is .


Let us assume r is the radius of the circle.





⇒ r = √5


We know that the equation of the circle with centre (p, q) and having radius ‘r’ is given by:


⇒ (x - p)2 + (y - q)2 = r2


Now we substitute the corresponding values in the equation:



⇒ x2 + 4x + 4 + y2 - 4y + 4 = 5


⇒ x2 + y2 + 4x - 4y + 3 = 0


∴The equation of the circle is x2 + y2 + 4x - 4y + 3 = 0.



Question 11.

Find the equation of the circle which circumscribes the triangle formed by the lines x = 0, y = 0 and lx + my = 1.


Answer:

Given that we need to find the equation of the circle that circumscribes the triangle formed by the lines x = 0, y = 0, and lx + my = 1.



Let us assume A, B, C are the vertices of the triangle.


On solving the lines we get,


⇒ A = (0,0)


⇒ B =


⇒ C =


We have the circle passing through the points A(0,0), B and C.


We know that the standard form of the equation of the circle is given by:


⇒ x2 + y2 + 2ax + 2by + c = 0 ..... (1)


Substituting A(0,0) in (1), we get,


⇒ 02 + 02 + 2a(0) + 2b(0) + c = 0


⇒ c = 0 ..... (2)


Substituting B in (1), we get,




⇒ cm2 + 2mb + 1 = 0 ..... - (3)


Substituting C in (1), we get,




⇒ cl2 + 2al + 1 = 0 ..... (4)


On solving (2), (3) and (4) we get,



Substituting these values in (1), we get




∴The equation of the circle is .



Question 12.

Find the equations of the circles which pass through the origin and cut off equal chords of √2 units from the lines y = x and y = - x.


Answer:

We need to find the equations of the circles which pass through the origin and having chords of units from the lines y = x and y = - x.



From the figure, we can see that the chords of length √2 units from origin O exists at points A(1,1), B(1, - 1), C(- 1,1) and D(- 1, - 1).


Let us find the distances AB, AC, BD, CD.


We know that distance between two points (x1,y1) and (x2,y2) is .




⇒ AB = 2


Similarly AD = BC = CD = 2 units.


We have got right-angled triangles OAB, OAD, OBC, OCD.


We need to find the circle that circumscribes these circles.


We know that the circumcentre of a right-angled triangle is the mid - point of the hypotenuse.


⇒ Circumcentre (C1) of OAB =


⇒ C1 = (1,0)


The radius of the circle is half of the length of the hypotenuse.



⇒ r1 = 1 units.


We know that the equation of the circle with centre (p, q) and having radius ‘r’ is given by:


⇒ (x - p)2 + (y - q)2 = r2


Now we substitute the corresponding values in the equation to get the circle’s equation for ΔOAB:



⇒ x2 - 2x + 1 + y2 = 1


⇒ x2 + y2 - 2x = 0


⇒ Circumcentre (C2) of OAD =


⇒ C2 = (0,1)


The radius of the circle is half of the length of the hypotenuse.



⇒ r2 = 1 units.


We know that the equation of the circle with centre (p, q) and having radius ‘r’ is given by:


⇒ (x - p)2 + (y - q)2 = r2


Now we substitute the corresponding values in the equation to get the circle’s equation for ΔOAD:



⇒ x2 + y2 - 2y + 1 = 1


⇒ x2 + y2 - 2y = 0


⇒ Circumcentre (C3) of OBC =


⇒ C3 = (0, - 1)


The radius of the circle is half of the length of the hypotenuse.



⇒ r3 = 1 units.


We know that the equation of the circle with centre (p, q) and having radius ‘r’ is given by:


⇒ (x - p)2 + (y - q)2 = r2


Now we substitute the corresponding values in the equation to get the circle’s equation for ΔOBC:



⇒ x2 + y2 + 2y + 1 = 1


⇒ x2 + y2 + 2y = 0


⇒ Circumcentre (C4) of OCD =


⇒ C4 = (- 1,0)


The radius of the circle is half of the length of the hypotenuse.



⇒ r4 = 1 units.


We know that the equation of the circle with centre (p, q) and having radius ‘r’ is given by:


⇒ (x - p)2 + (y - q)2 = r2


Now we substitute the corresponding values in the equation to get the circle’s equation for ΔOAC:



⇒ x2 + 2x + 1 + y2 = 1


⇒ x2 + y2 + 2x = 0


∴The equation of the circles are x2 + y2 - 2x = 0, x2 + y2 - 2y = 0, x2 + y2 + 2y = 0 and x2 + y2 + 2x = 0.




Very Short Answer
Question 1.

Write the length of the intercept made by the circle x2 + y2 + 2x – 4y – 5 = 0 on y - axis.


Answer:

Given that we need to find the length of the intercept made by the circle x2 + y2 + 2x - 4y - 5 = 0 on the y - axis.



For finding the length of intercept we first need to find the intercept made by the circle with the y - axis.


We know that on the y - axis, the value of x is 0.


By substituting x = 0 in the circle equation we get,


⇒ 02 + y2 + 2(0) - 4y - 5 = 0


⇒ y2 - 4y - 5 = 0


⇒ y2 - 5y + y - 5 = 0


⇒ y(y - 5) + 1(y - 5) = 0


⇒ (y + 1)(y - 5) = 0


⇒ y + 1 = 0 or y - 5 = 0


⇒ y = - 1 or y = 5.


The points intersected by the circle are A(- 1,0) and B(5,0).


We know the length of the intercept is AB.




⇒ AB = 6.


∴The length of the intercept is 6.



Question 2.

Write the coordinates of the centre of the circle passing through (0, 0), (4, 0) and (0, - 6).


Answer:

Given that we need to find the centre of the circle passing through O(0,0), A(4,0) and B(0, - 6).



Let us find the lengths of the triangle OAB.


We know that the distance between the points (x1,y1) and (x2,y2) is .




⇒ OA = 4




⇒ OB = 6




⇒ AB = √52


Consider OA2 + OB2,


⇒ OA2 + OB2 = 42 + 62


⇒ OA2 + OB2 = 16 + 36


⇒ OA2 + OB2 = 52



⇒ OA2 + OB2 = AB2


We got triangle OAB is a right-angled triangle with AB as the hypotenuse.


We know that the circumcentre of the right-angled triangle is the midpoint of the hypotenuse.


⇒ Centre =


⇒ Centre = (2, - 3)


∴The coordinates of the centre is (2, - 3).



Question 3.

Write the area of the circle passing through (- 2, 6) and having its centre at (1, 2).


Answer:

We need to find the area of the circle passing through (- 2,6) and having a centre at (1,2).



We know that the radius of the circle is the distance between the centre and any point on the radius. So, we find the radius of the circle.


We know that the distance between the two points (x1,y1) and (x2,y2) is .


Let us assume r is the radius of the circle.





⇒ r = √25


⇒ r = 5


We know that the area of the circle is r2.


⇒ Area(A) = π(5)2


⇒ A = π(25)


∴The area of the circle is 25π.



Question 4.

If the abscissae and ordinates of two points P and Q are roots of the equations x2 + 2ax – b2 = 0 and x2 + 2px – q2 = 0 respectively, then write the equation of the circle with PQ as diameter.


Answer:

Given that points, we need to find the equation of the circle whose ends of the diameter are P and Q.


It is also told that the abscissae of two points P and Q are the roots of x2 + 2ax - b2 = 0 and ordinates are the roots of x2 + 2px - q2 = 0.


Let us first find the roots of each quadratic equation.


For x2 + 2ax - b2 = 0




..... (1)


For x2 + 2px - q2 = 0




..... (2)


From (1) and (2) we get,




We know that the centre is the mid - point of the diameter.


⇒ Centre(C) =


⇒ C = (- a, - p)


We have a circle with centre (- a, - p) and passing through the point .


We know that the radius of the circle is the distance between the centre and any point on the radius. So, we find the radius of the circle.


We know that the distance between the two points (x1,y1) and (x2,y2) is .


Let us assume r is the radius of the circle.





We know that the equation of the circle with centre (p, q) and having radius ‘r’ is given by:


⇒ (x - p)2 + (y - q)2 = r2


Now we substitute the corresponding values in the equation:



⇒ x2 + 2ax + a2 + y2 + 2py + p2 = a2 + b2 + p2 + q2


⇒ x2 + y2 + 2ax + 2py - b2 - q2 = 0


∴The equation of the circle is x2 + y2 + 2ax + 2py - b2 - q2 = 0.



Question 5.

Write the equation of the unit circle concentric with x2 + y2 – 8x + 4y – 8 = 0.


Answer:

Given that we need to find the equation of the circle which is concentric with x2 + y2 - 8x + 4y - 8 = 0 and having a unit radius.



We know that concentric circles will have the same centre.


Let us assume the concentric circle be x2 + y2 - 8x + 4y + c = 0. ...... (ii)


We know that for a circle x2 + y2 + 2ax + 2by + c = 0 ...... (1)


⇒ Centre = (- a, - b)


⇒ Radius =


We know that unit circle has radius 1.



⇒ 1 = 16 + 4 - c


⇒ c = 19


∴ The equation of the concentric circle is x2 + y2 - 8x + 4y + 19 = 0.



Question 6.

If the radius of the circle x2 + y2 + ax + (1 – a) y + 5 = 0 does not exceed 5, write the number of integral values a.


Answer:

Given equation of circle is x2 + y2 + ax + (1 - a)y + 5 = 0.


We know that for a circle x2 + y2 + 2ax + 2by + c = 0


⇒ Centre = (- a, - b)


⇒ Radius =


We need to find the values of ‘a’ such that the radius of the given circle does not exceed 5.


We know that the radius of a circle cannot be less than 0.


Let ‘r’ be the radius of the given circle.


⇒ 0≤r≤5




⇒ 0≤2a2 - 2a - 19≤100


By trial and error method we get the set of values of ‘a’ as [ - 7.2, 8.2].


∴The integral values of ‘a’ are - 7, - 6, - 5, - 4, - 3, - 2, - 1,0,1,2,3,4,5,6,7,8.



Question 7.

Write the equation of the circle passing through (3, 4) and touching y-axis at the origin.


Answer:

Given: Circle passes through (3, 4) and touch y-axis at origin, this means it also passes though origin O(0, 0).


To Find: Equation of Circle


General equation of Circle: x2 + y2 + 2gx + 2fy + c = 0


As circle passes through (0, 0). This point will satisfy the equation.


Therefore,


(0)2 + (0)2 + 2g(0) + 2f(0) + c = 0


c = 0


Now, we have the equation of circle as,


x2 + y2 + 2gx + 2fy = 0


Now, since the centre is on the x-axis, y coordinate of centre = 0. i.e. f = 0.


Therefore,


x2 + y2 + 2gx = 0


It is also given that this circle passes through (3, 4). So,


(3)2 + (4)2 + 2g(3) = 0


9 + 16 + 6g = 0


25 + 6g = 0


6g = -25



Hence, we have the equation as,




Question 8.

If the line y = mx does not intersect the circle (x + 10)2 + (y + 10)2 = 180, then write the set of values taken by m.


Answer:

Given that we find the values of ‘m’ such that y = mx does not intersect the circle (x + 10)2 + (y + 10)2 = 180


The line does not intersect the circle if the perpendicular distance between the centre and the line is greater than the radius of the circle.


Here the centre and radius of the circle is (- 10, - 10) and √180.


We know that the perpendicular distance from the point (x1,y1)to the line ax + by + c = 0 is .





⇒ 100 - 200m + 100m2>180 + 180m2


⇒ 80m2 + 200m + 80<0


⇒ 2m2 + 5m + 2<0






We know that the solution set for the inequality (x - a)(x - b)<0 is (a, b) for b>a.



∴The solution set for ‘m’ is .



Question 9.

Write the coordinates of the centre of the circle inscribed in the square formed by the lines x = 2, x = 6, y = 5 and y = 9.


Answer:

Given that we need to find the centre of the circle inscribed square.



It is also told that x = 2, x = 6, y = 5 and y = 9 are the sides of a square.


Let us assume A,B,C,D are the vertices of the square. On solving the lines we get the vertices as:


⇒ A = (2,5)


⇒ B = (6,5)


⇒ C = (6,9)


⇒ D = (2,9)


Since the diagonal of the square is diameter of circle as the circle circumscribes the square.


So, taking any diagonal as diameter gives the same centre of the circle.


Let us assume diagonal AC as the diameter.


We know that centre is the mid - point of the diameter.


⇒ Centre(C) =


⇒ C = (4,7)


∴The coordinates of the centre is (4,7).




Mcq
Question 1.

If the equation of a circle is λx2 + (2λ – 3)y2 – 4x + 6y – 1 = 0, then the coordinates of centre are
A.

B.

C.

D.


Answer:

Given that the equation of the circle is:


⇒ λx2 + (2λ - 3)y2 - 4x + 6y - 1 = 0 ..... (1)


Comparing with the standard equation of circle:


⇒ x2 + y2 + 2ax + 2by + c = 0


We get


⇒ λ = 2λ - 3


⇒ 2λ - λ = 3


⇒ λ = 3


Substituting λ value in (1), we get


⇒ 3x2 + (2(3) - 3)y2 - 4x + 6y - 1 = 0


⇒ 3x2 + 3y2 - 4x + 6y - 1 = 0



We know that for a circle x2 + y2 + 2ax + 2by + c = 0


⇒ Centre = (- a, - b)


⇒ Radius =


⇒ Centre(C) =



∴The correct option is (b)


Question 2.

If 2x2 + λxy + 2y2 + (λ – 4) x + 6y – 5 = 0 is the equation of a circle, then its radius is
A.

B.

C.

D. none of the above


Answer:

Given that the equation of the circle is:


⇒ 2x2 + λxy + 2y2 + (λ - 4)x + 6y - 5 = 0 ..... (1)


Comparing with the standard equation of circle:


⇒ x2 + y2 + 2ax + 2by + c = 0


We get


⇒ λ = 0


Substituting λ value in (1), we get


⇒ 2x2 + 0xy + 2y2 + (0 - 4)x + 6y - 5 = 0


⇒ 2x2 + 2y2 - 4x + 6y - 5 = 0



We know that for a circle x2 + y2 + 2ax + 2by + c = 0


⇒ Centre = (- a, - b)


⇒ Radius =


⇒ Radius (r) =





∴The correct option is (d)


Question 3.

Mark the correct alternatives in each of the following :

The equation x2 + y2 + 2x – 4y + 5 = 0 represents
A. A point

B. A pair of straight lines

C. A circle of non - zero radius

D. None of these


Answer:

Given equation is:


⇒ x2 + y2 + 2x - 4y + 5 = 0



We know that for a pair of straight lines ax2 + 2hxy + by2 + 2gx + 2fy + c = 0, the conditions to be satisfied is abc + 2fgh - af2 - bg2 - cf2 = 0, h2≥ab, g2≥ca and f2≥bc.


Here we can see that h = 0 a = b = 1


⇒ h2 = 02 = 0


⇒ ab = 1.1 = 1


⇒ h2≤ab. So, the equation doesn’t represent a pair of straight lines.


⇒ x2 + y2 + 2x - 4y + 5 = 0


⇒ (x2 + 2x + 1) + (y2 - 4y + 4) = 0


⇒ (x + 1)2 + (y - 2)2 = 0


The equation represents circle with zero radius. This is a point circle.


∴The correct option is (a).


Question 4.

If the equation (4a – 3) x2 + ay2 + 6x – 2y + 2 = 0 represents a circle, then its centre is
A. (3, - 1)

B. (3,1)

C. (- 3,1)

D. none of these


Answer:

Given that the equation of the circle is:


⇒ (4a - 3)x2 + ay2 + 6x - 2y + 2 = 0 ..... (1)


Comparing with the standard equation of circle:


⇒ x2 + y2 + 2ax + 2by + c = 0


We get


⇒ 4a - 3 = a


⇒ 4a - a = 3


⇒ a = 1


Substituting a value in (1), we get


⇒ (4(1) - 3)x2 + 1y2 + 6x - 2y + 2 = 0


⇒ x2 + y2 + 6x - 2y + 2 = 0


We know that for a circle x2 + y2 + 2ax + 2by + c = 0


⇒ Centre = (- a, - b)


⇒ Radius =


⇒ Centre(C) =



∴The correct option is (c)


Question 5.

The radius of the circle represented by equation 3x2 + 3y2 + λxy + 9x + (λ – 6) y + 3 = 0 is
A.

B.

C.

D. none of these


Answer:

Given that the equation of the circle is:


⇒ 3x2 + 3y2 + λxy + 9x + (λ - 6)y + 3 = 0 ...... (1)


Comparing with the standard equation of circle:


⇒ x2 + y2 + 2ax + 2by + c = 0


We get


⇒ λ = 0


Substituting λ value in (1), we get


⇒ 3x2 + 3y2 + 0xy + 9x + (0 - 6)y + 3 = 0


⇒ 3x2 + 3y2 + 9x - 6y + 3 = 0


⇒ x2 + y2 + 3x - 2y + 1 = 0


We know that for a circle x2 + y2 + 2ax + 2by + c = 0


⇒ Centre = (- a, - b)


⇒ Radius =


⇒ Radius (r) =





∴The correct option is (a)


Question 6.

The number of integral values of λ for which the equation x2 + y2 + λx + (1 – λ) y + 5 = 0 is the equation of a circle whose radius cannot exceed 5, is
A. 14

B. 18

C. 16

D. none of these


Answer:

Given equation of circle is x2 + y2 + λx + (1 - λ)y + 5 = 0.


We know that for a circle x2 + y2 + 2ax + 2by + c = 0


⇒ Centre = (- a, - b)


⇒ Radius =


We need to find the values of ‘a’ such that the radius of the given circle does not exceed 5.


We know that the radius of a circle cannot be less than 0.


Let ‘r’ be the radius of the given circle.


⇒ 0≤r≤5




⇒ 0≤2λ2 - 2λ - 19≤100


By trial and error method we get the set of values of ‘λ’ as [ - 7.2, 8.2].


The integral values of ‘λ’ are - 7, - 6, - 5, - 4, - 3, - 2, - 1,0,1,2,3,4,5 ,6,7,8.


The no. of values of values of ‘λ’ is 16.


∴The correct option is (c)


Question 7.

The equation of the circle passing through the point (1, 1) and having two diameters along the pair of lines x2 – y2 – 2x – 3 = 0, is
A. x2 + y2 - 2x - 4y + 4 = 0

B. x2 + y2 + 2x + 4y - 4 = 0

C. x2 + y2 - 2x + 4y + 4 = 0

D. none of these


Answer:

Given that we need to find the equation of the circle passing through the point (1,1) and having two diameters along the pair of lines x2 - y2 - 2x - 3 = 0.


We know that the point of intersection of the pair of straight lines ax2 + 2hxy + by2 + 2gx + 2fy + c = 0 is .


Let us assume the intersection point of the pair of lines be ‘O’.




⇒ O = (1,0)


We have a circle with centre (1,0) and passing through the point (1,1).


We know that the radius of the circle is the distance between the centre and any point on the radius. So, we find the radius of the circle.


We know that the distance between the two points (x1,y1) and (x2,y2) is .


Let us assume r is the radius of the circle.





⇒ r = 1


We know that the equation of the circle with centre (p, q) and having radius ‘r’ is given by:


⇒ (x - p)2 + (y - q)2 = r2


Now we substitute the corresponding values in the equation:



⇒ x2 - 2x + 1 + y2 = 1


⇒ x2 + y2 - 2x = 0


∴The correct option is (d).


Question 8.

If the centroid of an equilateral triangle is (1, 1) and its one vertex is (- 1, 2), then the equation of its circumcircle is
A. x2 + y2 - 2x - 2y - 3 = 0

B. x2 + y2 + 2x - 2y - 3 = 0

C. x2 + y2 + 2x + 2y - 3 = 0

D. none of the these


Answer:

Given that we need to find the equation of circumcircle of an equilateral triangle whose centroid is (1,1) and one of its vertex is (- 1,2).



We know that in an equilateral triangle, the centroid and circumcentre coincides and circumcircle passes through all the vertices of the triangle.


We have a circle with centre (1,1) and passing through the point (- 1,2).


We know that the radius of the circle is the distance between the centre and any point on the radius. So, we find the radius of the circle.


We know that the distance between the two points (x1,y1) and (x2,y2) is .


Let us assume r is the radius of the circle.






We know that the equation of the circle with centre (p, q) and having radius ‘r’ is given by:


⇒ (x - p)2 + (y - q)2 = r2


Now we substitute the corresponding values in the equation:



⇒ x2 - 2x + 1 + y2 - 2y + 1 = 5


⇒ x2 + y2 - 2x - 2y - 3 = 0


∴The correct option is (a).


Question 9.

If the point (2, k) lies outside the circles x2 + y2 + x – 2y – 14 = 0 and x2 + y2 = 13 then k lies in the interval
A. (- 3, - 2) ∪ (3, 4)

B. - 3, 4

C. (- ∞, - 3) ∪ (4, ∞)

D. (- ∞, - 2) ∪ (3, ∞)


Answer:

Given that point (2,k) lies outsides the circles x2 + y2 + x - 2y - 14 = 0 and x2 + y2 = 13.



We know that for a point (a, b) to lie outside the circle S, the condition to be satisfies is S11>0.


Applying S11>0 for 1st circle,


⇒ 22 + k2 + 2 - 2(k) - 14>0


⇒ 4 + k2 - 2k - 12>0


⇒ k2 - 2k - 8>0


⇒ k2 - 4k + 2k - 8>0


⇒ k(k - 4) + 2(k - 4)>0


⇒ (k + 2)(k - 4)>0


We know that the solution set of (x - a)(x - b)>0 for b>a is (- ∞,a)∪(b,∞).


The solution set for k is (- ∞, - 2)∪(4,∞) ..... (1)


Applying S11>0 for 2nd circle,


⇒ 22 + k2 - 13>0


⇒ k2 + 4 - 13>0


⇒ k2 - 9>0


⇒ (k - 3)(k + 3)>0


The solution set for k is (- ∞, - 3)∪(3,∞) ...... (2)


The resultant solution set for k is the intersection of (1) and (2).


⇒ k(((- ∞, - 2)∪(4,∞))∩((- ∞, - 3)∪(3,∞)))


⇒ k((- ∞, - 3)∪(4,∞))


∴The correct option is (c).


Question 10.

If the point (λ, λ + 1) lies inside the region bounded by the curve and y - axis, then λ belongs to the interval
A. (- 1,3)

B. (- 4,2)

C. (- ∞, - 4)∪(3,∞)

D. none of these


Answer:

Given that the point (λ, λ + 1) lies inside the region bounded by the curve and y - axis.



The curve is rewritten as,


⇒ x2 = 25 - y2


⇒ x2 + y2 = 25


⇒ S = x2 + y2 - 25


We know that for a point (a, b) to lie inside the circle S, the condition to be satisfied is S11<0.


Applying S11<0 for 1st circle,


⇒ λ2 + (λ + 1)2 - 25<0


⇒ λ2 + λ2 + 2λ + 1 - 25<0


⇒ 2λ2 + 2λ - 24<0


⇒ λ2 + λ - 12<0


⇒ λ2 + 4λ - 3λ - 12<0


⇒ λ(λ + 4) - 3(λ + 4)<0


⇒ (λ - 3)(λ + 4)<0


We know that the solution set of (x - a)(x - b)<0 for b>a is (a,b).


The solution set for λ is (- 4,3) ...... (1)


Since the point lies inside y - axis λ + 1>0


⇒ λ> - 1 ..... (2)


The resultant solution set is the intersection of (1) and (2).


⇒ λ((- 4,3)∩(- 1,∞))


⇒ λ(- 1,3)


∴ The correct option is (a).


Question 11.

The equation of the incircle formed by the coordinate axes and the line 4x + 3y = 6 is
A. x2 + y2 - 6x - 6y + 9 = 0

B. 4(x2 + y2 - x - y) + 1 = 0

C. 4(x2 + y2 + x + y) + 1 = 0

D. None of these


Answer:

Given that we need to find the equation of the incircle formed by the coordinate axes and the line 4x + 3y = 6.



Let us find the vertices of triangle.


We know that the axes meet at origin O.


When the line meets x - axis, the value of ‘y’ is 0.


⇒ 4x + 3(0) = 6




The point is A.


When the line meets y - axis, the value of ‘x’ is 0.


⇒ 4(0) + 3y = 6



⇒ y = 2


The point is B(0,2).


Let us find the length of sides of the triangle.


We know that the distance between two points (x1,y1) and (x2,y2) is .







⇒ OB = b = 2






We know that incentre of a triangle is given by:






We have x = 0 as tangent to this circle.


We know that the perpendicular distance between the circle and centre is equal to the radius of the circle.


We know that the perpendicular distance between from the point (x1,y1) to the line ax + by + c = 0 is



.


We know that the equation of the circle with centre (p, q) and having radius ‘r’ is given by:


⇒ (x - p)2 + (y - q)2 = r2


Now we substitute the corresponding values in the equation:




⇒ 4(x2 + y2 - x - y) + 1 = 0


∴The correct option is (b).


Question 12.

If the circles x2 + y2 = 9 and x2 + y2 + 8y + c = 0 touch each other, then c is equal to
A. 15

B. - 15

C. 16

D. – 16


Answer:

Given that the circles x2 + y2 = 9 and x2 + y2 + 8y + c = 0 touch each other externally(assumed).



We need to find the value of c.


We know that if the circles touch each other externally, the distance between the centres is equal to the sum of the radii of two circles.


We know that for a circle x2 + y2 + 2ax + 2by + c = 0


⇒ Centre = (- a, - b)


⇒ Radius =


For x2 + y2 = 9


⇒ Centre(C1) = (0,0)


⇒ Radius(r1) =


⇒ r1 = 3


For x2 + y2 + 8y + c = 0


⇒ Centre(C2) =


⇒ C2 = (0, - 4)


⇒ Radius(r2) =



We have C1C2 = r1 + r2





⇒ 1 = 16 - c


⇒ c = 16 - 1


⇒ c = 15


∴The correct option is (a).


Question 13.

If the circle x2 + y2 + 2ax + 8y + 16 = 0 touches x - axis, then the value of a is
A. ±16

B. ±4

C. ±8

D. ±1


Answer:

Given that the circle x2 + y2 + 2ax + 8y + 16 = 0 touches x - axis. We need to find the value of a.



We know that the value of y on the x - axis is 0.


⇒ x2 + (0)2 + 2ax + 8(0) + 16 = 0


⇒ x2 + 2ax + 16 = 0


The quadratic equation will have similar roots if the circle touches the x - axis.


We know that for the quadratic equation ax2 + bx + c = 0 the condition to be satisfied for the equal roots is b2 - 4ac = 0


⇒ (2a)2 - 4(1)(16) = 0


⇒ 4a2 - 64 = 0


⇒ a2 = 16


⇒ a = ±4


∴The correct option is (b).


Question 14.

The equation of a circle with radius 5 and touching both the coordinate axes is
A. x2 + y2±10x±10x + 5 = 0

B. x2 + y2±10x±10y = 0

C. x2 + y2±10x±10y + 25 = 0

D. x2 + y2±10x±10y + 51 = 0


Answer:

Given that the circle having radius 5 touches both the coordinate axes.



Let us assume the circle touches the co - ordinate axes at (a,0) and (0,a). Then the circle will have the centre at (a, a) and radius |a|.


It is given that the radius is 5 units.


The centre of the circle is (±5,±5).


We know that the equation of the circle with centre (p, q) and having radius ‘r’ is given by:


⇒ (x - p)2 + (y - q)2 = r2


Now we substitute the corresponding values in the equation:


⇒ (x±5)2 + (y±5)2 = (5)2


⇒ x2±10x + 25 + y2±10y + 25 = 25


⇒ x2 + y2±10x±10y + 25 = 0


∴The correct option is (c).


Question 15.

The equation of the circle passing through the origin which cuts off intercept of length 6 and 8 from the axes is
A. x2 + y2 - 12x - 16y = 0

B. x2 + y2 + 12x + 16y = 0

C. x2 + y2 + 7x + 8y = 0

D. x2 + y2 - 6x - 8y = 0


Answer:

Given that we need to find the equation of the circle passing through origin and cuts off intercepts 6 and 8 from x and y - axes.



Since the circle is having intercept a from x - axis the circle must pass through (6,0) and (- 6,0) as it already passes through the origin.


Since the circle is having intercept 8 from x - axis the circle must pass through (0,8) and (0, - 8) as it already passes through the origin.


Let us assume the circle passing through the points O(0,0), A(6,0) and B(0,8).


We know that the standard form of the equation of the circle is given by:


⇒ x2 + y2 + 2fx + 2gy + c = 0 ..... (1)


Substituting O(0,0) in (1), we get,


⇒ 02 + 02 + 2f(0) + 2g(0) + c = 0


⇒ c = 0 ..... (2)


Substituting A(6,0) in (1), we get,


⇒ 62 + 02 + 2f(6) + 2g(0) + c = 0


⇒ 36 + 12f + c = 0 ..... (3)


Substituting B(0,8) in (1), we get,


⇒ 02 + 82 + 2f(0) + 2g(8) + c = 0


⇒ 64 + 16g + c = 0 ..... (4)


On solving (2), (3) and (4) we get,


⇒ f = - 3, b = - 4 and c = 0


Substituting these values in (1), we get


⇒ x + y2 + 2(- 3)x + 2(- 4)y + 0 = 0


⇒ x2 + y2 - 6x - 8y = 0


Similarly, we get the equation x2 + y2 + 6x + 8y = 0 for the circle passing through the points (0,0), (- 6,0), (0, - 8), x2 + y2 + 6x - 8y = 0 for the circle passing through the points (0,0), (- 6,0), (0,8) x2 + y2 - 6x + 8y = 0 for the circle passing through the points (0,0), (6,0), (0, - 8) .


∴The equations of the circles are x2 + y2±6x±8y = 0.


∴The correct options are (d).


Question 16.

The equation of the circle concentric with x2 + y2 – 3x + 4y – c = 0 and passing through (- 1, - 2) is
A. x2 + y2 - 3x + 4y - 1 = 0

B. x2 + y2 - 3x + 4y = 0

C. x2 + y2 - 3x + 4y + 2 = 0

D. none of these


Answer:

Given that we need to find the equation of the circle which is concentric with x2 + y2 - 3x + 4y - c = 0 and passing through (- 1, - 2).



We know that concentric circles will have same centre.


Let us assume the concentric circle be x2 + y2 - 3x + 4y + d = 0. ..... - (ii)


Substituting (- 1, - 2) in (ii) we get


⇒ (- 1)2 + (- 2)2 - 3(- 1) + 4(- 2) + d = 0


⇒ 1 + 4 + 3 - 8 + d = 0


⇒ d = 0


Substituting value of d in (ii) we get


⇒ x2 + y2 - 3x + 4y = 0


∴ The correct option is (b).


Question 17.

The circle x2 + y2 + 2gx + 2 fy + c = 0 does not intersect x - axis, if
A. g2<c

B. g2>c

C. g2>2c

D. none of these


Answer:

Given that the circle x2 + y2 + 2gx + 2fy + c = 0 does not intersect x - axis. We need to find the relation between g and c.


We know that the value of y on the x - axis is 0.


⇒ x2 + (0)2 + 2gx + 2f(0) + c = 0


⇒ x2 + 2gx + c = 0


The quadratic equation will have no roots if the circle does not intersect the x - axis.


We know that for the quadratic equation ax2 + bx + c = 0 the condition to be satisfied for the equal roots is b2 - 4ac<0


⇒ (2g)2 - 4(1)(c)<0


⇒ 4(g2 - c)<0


⇒ g2 - c<0


⇒ g2<c


∴The correct option is (a).


Question 18.

The area of an equilateral triangle inscribed in the circle x2 + y2 – 6x – 8y – 25 = 0 is
A.

B. 25 π

C. 50 π- 100

D. none of these


Answer:

We need to find the area of the equilateral triangle that is inscribed in the circle x2 + y2 - 6x - 8y - 25 = 0.



We know that for a circle x2 + y2 + 2ax + 2by + c = 0


⇒ Centre = (- a, - b)


⇒ Radius =


For x2 + y2 - 6x - 8y - 25 = 0


⇒ Radius(r1) =





From the figure we can see that,





We know that area of the equilateral triangle with side length ‘a’ is






.


∴The correct option is (a).


Question 19.

The equation of the circle which touches the axes of coordinates and the line and whose centres lie in the first quadrant is x2 + y2 – 2cx – 2cy + c2 = 0, where c is equal to
A. 4

B. 2

C. 3

D. 6


Answer:

Given that the equation of the circle which touches the axes of coordinates and the line and whose centres lie in the first quadrant is x2 + y2 - 2cx - 2cy + c2 = 0. We need to find the value of c.



We know that for a circle x2 + y2 + 2ax + 2by + c = 0


⇒ Centre = (- a, - b)


⇒ Radius =


For x2 + y2 - 2cx - 2cy + c2 = 0


⇒ Centre(C) =


⇒ C = (c,c)


⇒ Radius(r) =



⇒ r = c


We have touching the circle.


We know that the perpendicular distance between the circle and centre is equal to the radius of the circle.


We know that the perpendicular distance between from the point (x1,y1) to the line ax + by + c = 0 is



.







⇒ c2 - 7c + 6 = 0


⇒ c2 - 6c - c + 6 = 0


⇒ c(c - 6) - 1(c - 6) = 0


⇒ (c - 1)(c - 6) = 0


⇒ c - 1 = 0 or c - 6 = 0


⇒ c = 1 or c = 6


∴The correct option is (d).


Question 20.

If the circles x2 + y2 = a and x2 + y2 – 6x – 8y + 9 = 0, touch externally, then a =
A. 1

B. - 1

C. 21

D. 16


Answer:

Given that the circles x2 + y2 = a and x2 + y2 - 6x - 8y + 9 = 0 touch each other externally.



We need to find the value of a.


We know that if the circles touch each other externally, the distance between the centres is equal to the sum of the radii of two circles.


We know that for a circle x2 + y2 + 2ax + 2by + c = 0


⇒ Centre = (- a, - b)


⇒ Radius =


For x2 + y2 = a


⇒ Centre(C1) = (0,0)


⇒ Radius(r1) =



For x2 + y2 - 6x - 8y + 9 = 0


⇒ Centre(C2) =


⇒ C2 = (3,4)


⇒ Radius(r2) =




⇒ r2 = 4


We have C1C2 = r1 + r2








⇒ a = 1


∴The correct option is (a).


Question 21.

If (x, 3) and (3, 5) are the extremities of a diameter of a circle with centre at (2, y), then the values of x and y are
A. x = 3, y = 1

B. x = 4, y = 1

C. x = 8, y = 2

D. none of these


Answer:

Given that (x,3) and (3,5) are the extremities of a diameter of a circle with centre (2,y).



We know that centre is the midpoint of diameter.





⇒ x + 3 = 4 and y = 4


⇒ x = 1 and y = 4


∴The correct option is (d)


Question 22.

If (- 3, 2) lies on the circle x2 + y2 + 2gx + 2fy + c = 0 which is concentric with the circle x2 + y2 + 6x + 8y – 5 = 0, then c =
A. 11

B. - 11

C. 24

D. none of these


Answer:

Given that the point (- 3,2) lies on the circle x2 + y2 + 2gx + 2fy + c = 0 which is concentric with x2 + y2 + 6x + 8y - 5 = 0.



We know that concentric circles will have same centre.


Let us assume the concentric circle be x2 + y2 + 6x + 8y + d = 0. ..... - (ii)


Substituting (- 3,2) in (ii)


⇒ (- 3)2 + 22 + 6(- 3) + 8(2) + d = 0


⇒ 9 + 4 - 18 + 16 + d = 0


⇒ d = - 11


Substituting value of c in (ii) we get


⇒ x2 + y2 + 6x + 8y - 11 = 0


Comparing with x2 + y2 + 2gx + 2fy + c = 0 we get c = - 11


∴ The correct option is (b).


Question 23.

Equation of the diameter of the circle x2 + y2 – 2x + 4y = 0 which passes through the origin is
A. x + 2y = 0

B. x - 2y = 0

C. 2x + y = 0

D. 2x - y = 0


Answer:

Given that we need to find the equation of the diameter if the circle x2 + y2 - 2x + 4y = 0 which passes through origin.



We know that for a circle x2 + y2 + 2ax + 2by + c = 0


⇒ Centre = (- a, - b)


⇒ Radius =


For x2 + y2 - 2x + 4y = 0


⇒ Centre(C1) =


⇒ C1 = (1, - 2)


We know that the diameter of the circle passes through the centre.


We need to find the equation of the diameter passing through the points (1, - 2) and (0,0)


We know that the equation of the straight line passing through the points (x1,y1) and (x2,y2) is




⇒ y = - 2x


⇒ 2x + y = 0


∴The correct option is (c)


Question 24.

Equation of the circle through origin which cuts intercepts of length a and b on axes is
A. x2 + y2 + ax + by = 0

B. x2 + y2 - ax - by = 0

C. x2 + y2 + bx + ay = 0

D. none of these


Answer:

Given that we need to find the equation of the circle passing through origin and cuts off intercepts a and b from x and y - axes.


Since the circle is having intercept a from x - axis the circle must pass through (a,0) and (- a,0) as it already passes through the origin.


Since the circle is having intercept b from x - axis the circle must pass through (0,b) and (0, - b) as it already passes through the origin.


Let us assume the circle passing through the points O(0,0), A(a,0) and B(0,b).


We know that the standard form of the equation of the circle is given by:


⇒ x2 + y2 + 2fx + 2gy + c = 0 ..... (1)


Substituting O(0,0) in (1), we get,


⇒ 02 + 02 + 2f(0) + 2g(0) + c = 0


⇒ c = 0 ..... (2)


Substituting A(a,0) in (1), we get,


⇒ a2 + 02 + 2f(a) + 2g(0) + c = 0


⇒ a2 + 2fa + c = 0 ..... (3)


Substituting B(0,b) in (1), we get,


⇒ 02 + b2 + 2f(0) + 2g(b) + c = 0


⇒ b2 + 2gb + c = 0 ..... (4)


On solving (2), (3) and (4) we get,



Substituting these values in (1), we get



⇒ x2 + y2 - ax - by = 0


Similarly, we get the equation x2 + y2 + ax + by = 0 for the circle passing through the points (0,0), (- a,0), (0, - b).


∴The equations of the circles are x2 + y2±ax±by = 0.


∴The correct options are (a) and (b).


Question 25.

If the circles x2 + y2 + 2ax + c = 0 and x2 + y2 + 2by + c = 0 touch each other, then
A.

B.

C. a + b = 2c

D.


Answer:

Given that the circles x2 + y2 + 2ax + c = 0 and x2 + y2 + 2by + c = 0 touch each other externally (assumed).


We need to find the relation of a,b and c.


We know that if the circles touch each other externally, the distance between the centres is equal to the sum of the radii of two circles.


We know that for a circle x2 + y2 + 2ax + 2by + c = 0


⇒ Centre = (- a, - b)


⇒ Radius =


For x2 + y2 + 2ax + c = 0


⇒ Centre(C1) =


⇒ C1 = (- a,0)


⇒ Radius(r1) =



For x2 + y2 + 2by + c = 0


⇒ Centre(C2) =


⇒ C2 = (0, - b)


⇒ Radius(r2) =



We have C1C2 = r1 + r2








⇒ c2 = a2b2 - a2c - b2c + c2


⇒ a2b2 = a2c + b2c




∴The correct option is (a).